Sie sind auf Seite 1von 45

El gnero lrico

A continuacin puedes leer un poema de Blas de Otero (1916-1979) y otro de Gloria Fuertes (1918-1998). Ambos autores pertenecen a la poesa social, que se desarroll en Espaa a partir de los aos cincuenta.

ACTIVIDADE S

Djame
Me haces dao, Seor. Quita tu mano de encima. Djame con mi vaco, djame. Para abismo, con el mo tengo bastante. Oh Dios, si eres humano, compadcete ya, quita esa mano de encima. No me sirve. Me da fro y miedo. Si eres Dios, yo soy tan mo como t.Y a soberbio, yo te gano. Djame. Si pudiese yo matarte, como haces t, como haces t! Nos coges con las dos manos, nos ahogas. Matas no se sabe por qu. Quiero cortarte las manos. Esas manos que son trojes del hambre, y de los hombres que arrebatas.
Blas DE OTERO Poesa espaola contempornea (1939-1980), Alhambra

10

El planeta tierra
El planeta tierra debera llamarse planeta agua. En la tierra hay ms agua que cuerpo, en el cuerpo hay ms cuerpo que alma, en la tierra hay ms peces que aves, en las aves ms plumas que alas. En el verso hay ms sangre que tinta, en la tinta ms sombra que nada, en la nada hay ms algo que alga y ese algo se mueve y reluce y nace la palabra.
MATERIAL FOTOCOPIABLE / Oxford University Press Espaa, S. A.

10

Gloria FUERTES Poesa espaola contempornea (1939-1980), Alhambra

1 Justifica la adscripcin de estos dos textos al gnero lrico. 2 Localiza en el poema de Blas de Otero al emisor y al receptor y explica el contenido del mensaje. 3 Explica la mtrica de ambos textos: la rima, la medida de los versos, el agrupamiento estrfico y el tipo de poema.

4 Qu recurso expresivo abunda en el poema Djame? Localiza, adems, un apstrofe y un caso de similicadencia. 5 Cul es el mensaje del poema de Gloria Fuertes? Qu crees que significa: En el verso hay ms sangre que tinta? 6 Qu recurso expresivo de repeticin predomina en el segundo poema?

ACTIVIDADE S

8
El gnero narrativo
El siguiente relato pertenece al escritor argentino Jorge Luis Borges (1899-1986), quien ha destacado tanto por sus cuentos como por sus poemas. Entre sus obras narrativas se encuentran El jardn de los senderos que se bifurcan, El Sur, El libro de arena y El Aleph, al que pertenece este cuento.

Los dos reyes y los dos laberintos


Cuentan los hombres dignos de fe (pero Al sabe ms) que en los primeros das hubo un rey de las islas de Babilonia que congreg a sus arquitectos y magos y les mand construir un laberinto tan perplejo y sutil que los varones ms prudentes no se aventuraban a entrar, y los que entraban se perdan. Esa obra era un escndalo, porque la confusin y la maravilla son operaciones propias de Dios y no de los hombres. Con el andar del tiempo vino a su corte un rey de los rabes, y el rey de Babilonia (para hacer burla de la simplicidad de su husped) lo hizo entrar en el laberinto, donde vag afrentado y confundido hasta la declinacin de la tarde. Entonces implor socorro divino y dio con la puerta. Sus labios no profirieron queja ninguna, pero le dijo al rey de Babilonia que l en Arabia tena otro laberinto y que, si Dios era servido, se lo dara a conocer algn da. Luego regres a Arabia, junt a sus capitanes y sus alcaides y estrag los reinos de Babilonia con tan venturosa fortuna que derrib sus castillos, rompi sus gentes e hizo cautivo al mismo rey. Lo amarr encima de un caballo veloz y lo llev al desierto. Cabalgaron tres das, y le dijo: Oh, rey del tiempo y sustancia y cifra del siglo!, en Babilonia me quisiste perder en un laberinto de bronce con muchas escaleras, puertas y muros; ahora el Poderoso ha tenido a bien que te muestre el mo, donde no hay escaleras que subir, ni puertas que forzar, ni fatigosas galeras que recorrer, ni muros que te veden el paso. Luego le desat las ligaduras y lo abandon en mitad del desierto, donde muri de hambre y de sed. La gloria sea con Aquel que no muere.
Jorge Luis BORGES Obras completas, Emec

1 Justifica por qu podemos afirmar que estamos en presencia de un texto narrativo. 2 A qu subgnero narrativo pertenece este texto de Jorge Luis Borges? Argumenta tu respuesta. 3 Resume el contenido de la historia narrada por Borges. 4 Explica los conceptos de tiempo de la historia y tiempo del discurso a partir de este texto.

5 Qu movimientos narrativos reconoces en el texto? 6 Caracteriza al narrador y su perspectiva. 7 Seala en el cuento ejemplos de discurso referencial, valorativo, universal, directo e indirecto. 8 En el discurso directo presente en el fragmento tambin puede observarse discurso potico; explica por medio de qu recurso expresivo se materializa.

MATERIAL FOTOCOPIABLE / Oxford University Press Espaa, S. A.

ACTIVIDADE S

El gnero dramtico
El siguiente fragmento pertenece a uno de los principales dramaturgos espaoles del siglo XX, Antonio Buero Vallejo. Entre sus principales obras se encuentran Historia de una escalera (1949), que abri un nuevo panorama en el drama de posguerra, En la ardiente oscuridad (1950), Casi un cuento de hadas (1953), Irene o el tesoro (1954), El concierto de San Ovidio (1962), El tragaluz (1967) y La fundacin (1974).

La confusin
ALBERTO.[] Se oye un bastn. JUANA.Tambin yo lo oigo. ( Todos atienden. Pausa. Por la derecha, tanteando el suelo con su bastn y con una expresin de vago susto, aparece IGNACIO. Es un muchacho delgaducho, serio y reconcentrado, con cierto desalio en su persona: el cuello de la camisa desabrochado, la corbata floja, el cabello peinado con ligereza. Viste de negro intemporalmente, durante toda la obra. Avanza unos pasos indeciso y se detiene.) LOLITA.Qu raro! (IGNACIO se estremece y retrocede un paso.) MIGUEL.Quin eres? (Temeroso, IGNACIO se vuelve para salir por donde entr. Despus, cambia de idea y sigue hacia la izquierda, rpido.) ANDRS.No contestas? (IGNACIO tropieza con el silln de JUANA. Tiende el brazo y ella coge su mano.) MIGUEL.(Levantndose.) Espera, hombre! No te marches. (Se acerca a palparle, mientras JUANA dice, inquieta:) JUANA.Me ha cogido la mano No le conozco. (IGNACIO la suelta y MIGUELN le sujeta por un brazo.) MIGUEL.Ni yo.
MATERIAL FOTOCOPIABLE / Oxford University Press Espaa, S. A.

LOLITA.(Riendo.) Te ha salido un competidor, Migueln. ESPERANZA.Un competidor? Un maestro! ALBERTO.Debe de ser algn gracioso del primer curso. MIGUEL.Dejdmelo a m. Qu has dicho que eres? IGNACIO.(Asustado.) Un ciego. MIGUEL.Oh, pobrecito, pobrecito! Quiere que le pase a la otra acera? (Los dems se desternillan.) Largo, idiota! Vete a rer de los de tu curso. ANDRS.Realmente la broma es de muy mal gusto. Anda, mrchate. (Lo empujan. IGNACIO retrocede hacia el proscenio.) IGNACIO.(Violento, quiz al borde del llanto.) Os digo que soy ciego! MIGUEL.Qu bien te has aprendido la palabrita! Largo! (Avanzan hacia l, amenazadores. ALBERTO se levanta tambin.) IGNACIO.Pero, es que no lo veis? MIGUEL.Cmo? (JUANA y CARLOS, que comentaban en voz baja el incidente, intervienen.) CARLOS.Creo que estamos cometiendo un error muy grande, amigos. l dice la verdad. Sentaos otra vez. MIGUEL.Atiza! CARLOS.(Acercndose con JUANA a IGNACIO.) Nosotros tambin somos ciegos, como t dices.
Antonio BUERO VALLEJO En la ardiente oscuridad, Magisterio Espaol

(ANDRS se levanta y se acerca tambin para cogerle por el otro brazo.) IGNACIO. (Con temor.) Djame. ANDRS.Qu buscas aqu? IGNACIO.Nada. Dejadme. Yo soy un pobre ciego.

1 Explica lo que ocurre en este fragmento de la obra En la ardiente oscuridad, de Antonio Buero Vallejo. 2 Justifica la adscripcin de esta obra al gnero dramtico. 3 En qu subgnero lo incluiras? Argumenta tu respuesta.

4 Diferencia en el fragmento el texto principal y el texto secundario. 5 A qu hacen referencia las acotaciones que aparecen? 6 Transforma el fragmento La confusin en un texto narrativo. Recuerda que puedes incluir dilogo en l.

Evaluacin 8

Los espritus de la tormenta, por Evelyn de Morgan.

Rima LII
Olas gigantes que os rompis bramando en las playas desiertas y remotas, envuelto entre la sbana de espumas, llevadme con vosotras! Rfagas de huracn que arrebatis del alto bosque las marchitas hojas, arrastrado en el ciego torbellino, llevadme con vosotras! Nubes de tempestad que rompe el rayo y en fuego encienden las sangrientas orlas, arrebatado entre la niebla oscura, llevadme con vosotras! Llevadme por piedad a donde el vrtigo con la razn me arranque la memoria. Por piedad! tengo miedo de quedarme con mi dolor a solas! 10

15

Gustavo Adolfo BCQUER Libro de los gorriones, Planeta

1. Justifica por qu la Rima LII es un texto literario y por


qu pertenece al gnero lrico.

7. Consulta el anexo de las pginas 352-353 de este libro


y seala los recursos de repeticin que se emplean en el texto de Bcquer.

2. Identifica al emisor y a los receptores del proceso comunicativo interno del poema y comenta sus caractersticas.

8. Reconoce en esa misma composicin otros recursos


expresivos.

3. Qu actitud ofrece la voz potica en esta composicin?


Explica por qu.

9. Qu rasgos debera presentar esta composicin de


Gustavo Adolfo Bcquer para pertenecer a cada uno de los otros dos gneros literarios?

4. Explica el contenido del mensaje en esta rima. 5. Qu tema desarrolla? Indica qu motivos aparecen. 6. Analiza la mtrica. Atiende a la medida de los versos, al
esquema y el tipo de rima y al agrupamiento en estrofas.

10. Redacta el comentario de la Rima LII del Libro de los


gorriones integrando todos los aspectos que has analizado en las respuestas anteriores.

ACTIVIDADE S

1. Lrica culta gallego-portuguesa. Las Cantigas de Santa Mara


Aunque se admite que el rey Alfonso X el Sabio es el autor de algunas cantigas, se supone que la mayora fueron creadas por un reducido grupo de escritores cortesanos. De las 420 Cantigas de Santa Mara, solo 64 son de carcter lrico; la mayor parte son narrativas y relatan milagros de la Virgen. Todas van acompaadas de melodas y de preciosas miniaturas que permiten conocer aspectos de la vida cotidiana de la poca. Adems de la Introduccin y los dos prlogos, el resto de las cantigas lricas son de loor o alabanza a la Virgen. Entre ellas predominan las de tipo hmnico, que celebran a Mara como auxiliadora o se refieren a festividades marianas y cristolgicas. En las Cantigas se registran cerca de 300 combinaciones mtricas distintas, de las cuales las ms irregulares corresponden a las de carcter lrico. Los versos varan entre las dos y las veinticuatro slabas, y entre ellos destaca el alejandrino. En cuanto a las estrofas empleadas, predomina el zjel.

Cantiga 10
ESTA DE LOOR DE SANTA MARA, COM FREMOSA E BOA E GRAN PODER Rosas das rosas e Fror das frores, Dona das donas, Sennor das sennores. Rosa de beldad e de parecer e Fror d alegria e de prazer, Dona en mui piadosa seer, Sennor en toller coitas e doores. Rosa das rosas e Fror das frores A tal Sennor dev ome muit amar, que de todo mal o pode guardar; e pode-ll os peccados perdo ar, que faz no mundo per maos sabores. Rosa das rosas e Fror das frores Devemo-la muit amar e servir, ca punna de nos guardar de falir; des i dos erros nos faz repentir, que nos fazemos come pecadores. Rosa das rosas e Fror das frores
MATERIAL FOTOCOPIABLE / Oxford University Press Espaa, S. A.

Rosa de las rosas y Flor de las flores, Seora de las seoras, Seor de los seores. Rosa de beldad y jucio y Flor de alegra y placer, Seora muy piadosa en su ser, Seor en quitar penas y dolores. Rosa de las rosas A tal Seor debe el hombre mucho amar, que de todo mal lo puede guardar, y puede perdonarle los pecados, que acta en el mundo contra los sinsabores. Rosa de las rosas La debemos amar mucho y servir, porque lucha por guardarnos de caer; de los errores nos hace arrepentir, que nosotros hacemos como pecadores. Rosa de las rosas Esta seora que tengo por Seor y de quien quiero ser trovador, si yo por fortuna puedo tener su amor, doy al demonio otros amores. Rosa de las rosas

10

10

15

15

Esta dona que tenno por Sennor e de que quero seer trobador, se eu per ren poss aver seu amor, dou ao demo os outros amores. Rosa das rosas e Fror das frores

20

20

Alfonso X EL SABIO Cantigas de Santa Mara, Castalia

Traduccin de los autores

1 Por qu pertenece a la lrica la Cantiga 10? Para contestar, consulta la UNIDAD 11 y justifica tu respuesta. 2 Explica cmo se presenta la figura de la Virgen en este poema mariano.

3 Analiza la mtrica y los recursos expresivos empleados en la cantiga. 4 Qu composiciones de la lrica culta pueden haber influido en las Cantigas? Para responder, apyate en ejemplos extrados de la Cantiga 10.

ACTIVIDADE S

9
La lrica primitiva popular
En la Pennsula, la lrica primitiva popular se manifiesta en tres tipos de composiciones: las jarchas, las cantigas de amigo y los villancicos. Repasa las caractersticas formales y temticas de estas producciones literarias y despus lee detenidamente los siguientes textos.

Texto 1
Non dormireyo, mamma, a rayo de maana. Bon Abu-l-Qasim, la facha de matrana. No dormir, madre, al rayar la maana. El buen Abu-l-Qasim, la cara de la aurora.

Texto 5
Mia irmana fremosa, treydes comigo a la igreja de Vig, u o mar salido: E miraremos las ondas! Mia irmana fremosa, treydes de grado a la igresa de Vig, u o mar levado: 5 E miraremos las ondas! A la igresa de Vig, u o mar salido, e verr i, mia madre, o meu amigo: E miraremos las ondas! A la igresa de Vig, u o mar levado, 10 e verr y, mia madre, o meu amado: E miraremos las ondas! (Invita a su hermana a visitar la iglesia de Vigo durante la marea alta, y dice a su madre que all ir su amigo.)
Antologa de la poesa gallego-portuguesa Alhambra

Texto 2
Quiero dormir y no puedo, que el amor me quita el sueo.

Texto 3
Qu far, mamma? Meu al-habib est ad yana. Qu har, madre? Mi amigo est en la puerta.

Texto 4
Con amores, mi madre, con amores me adorm.
Lrica espaola de tipo popular Ctedra

1 Identifica entre estos cinco poemas cules son jarchas. En qu aspecto o aspectos te has basado para reconocerlas? 2 Caracteriza el emisor, receptor y contenido del mensaje en cada una de ellas. 3 Analiza el nmero de versos y la rima empleada en estas composiciones. 4 En una de las jarchas figura una metfora; localzala y explica su valor. 5 Entre los poemas se incluyen tambin villancicos castellanos. Selalos y explica en qu te has basado para reconocerlos.

6 Establece el proceso de la comunicacin en esos poemas: emisor, receptor y contenido del mensaje. 7 Qu tipo de poema configura la cantiga de amigo? Para responder, consulta, si es necesario, el apndice (pginas 358 y 359) y la pgina 184 de tu libro de texto. 8 Establece las semejanzas de contenido entre el poema del texto 5, las jarchas y los villancicos; seala, igualmente, las semejanzas y diferencias formales entre estos tres tipos de composiciones de la lrica primitiva popular.

194 12.

La lrica medieval

MATERIAL FOTOCOPIABLE / Oxford University Press Espaa, S. A.

ACTIVIDADE S

La lrica del siglo XV


El siglo XV es una poca de eclosin de la poesa. El Romancero constituye un hito en nuestra tradicin literaria y crea una tipologa que permanece hasta la actualidad. La poesa culta de los cancioneros ser la fuente de la que bebern los poetas renacentistas.

10

Texto 1
Y pues vemos lo presente cmo en un punto se es ido y acabado, si juzgamos sabiamente, daremos lo non venido por pasado. No se engae nadie, no, pensando que a de durar lo que espera ms que dur lo que vio, porque todo ha de pasar por tal manera.

Texto 2
Tus casos falaces, Fortuna, cantamos, estados de gentes que giras y trocas, tus grandes discordias, tus firmezas pocas, y los que en tu rueda quejosos hallamos, hasta que al tiempo de ahora vengamos; de hechos pasados codicia mi pluma, y de los presentes, hacer breve suma; d fin Apolo, pues nos comenzamos.

10

Jorge MANRIQUE Poesa, Crtica

Juan DE MENA El laberinto de la Fortuna, Espasa Calpe

Texto 3
Ven, muerte, tan escondida que no te sienta conmigo, porque el gozo de contigo no me torne a dar la vida. Ven como rayo que hiere: que hasta que ha herido no se siente su ruido, por mejor herir do quiere. As sea tu venida; si no desde aqu me obligo que el gozo que habr contigo me dar de nuevo vida. 5

Texto 4
Vos me matis de tal suerte, con pena tan glorosa, que no s ms dulce cosa que los trances de mi muerte. Y de ella soy tan ufano, 5 tan penado y tan contento, que no trocar un tormento por mil bienes de otra mano. Y pues que quiso mi suerte darme pena gloriosa, 10 no quiero ms dulce cosa que los trances de mi muerte.
Poesa medieval castellana Taurus

Texto 5
Soy contento, vos servida, ser penado de tal suerte, que por vos quiero la muerte mas que no, sin vos, la vida. Quiero ms por vos tristura, siendo vuestro sin mudanza, que placer sin esperanza de enamorada ventura. No tengis la fe perdida pues la tengo yo tan fuerte, que por vos quiero la muerte mas que no, sin vos, la vida. 5

10

10

Poesa medieval castellana Taurus

Poesa medieval castellana Taurus

MATERIAL FOTOCOPIABLE / Oxford University Press Espaa, S. A.

1 Reconoce en el texto 1 la mtrica empleada por su autor, Jorge Manrique, en las Coplas por la muerte de su padre y seala en la estrofa otras caractersticas de esta obra. 2 Quin es el receptor del texto 2? Qu le dice la voz potica? Explica qu intencin manifiesta en los tres ltimos versos. 3 Reconoce la mtrica empleada por Juan de Mena en la obra a la que pertenece la estrofa del texto 2 y seala en ella otras caractersticas de esta obra.

4 A qu tipo de composiciones corresponden los otros tres textos? Explica su mtrica y su estructura. Texto 1 -+ Texto 2 -+ Texto 3 -+ 5 Explica brevemente el contenido de cada uno de los poemas reproducidos en esta pgina. 6 Seala en los textos 3, 4 y 5 los recursos expresivos caractersticos de este tipo de composiciones.

ACTIVIDADE S

10
El arte de los trovadores
El arte de los trovadores provenzales fue producto de un largo proceso de aprendizaje de potica, retrica y msica. Ya desde la primera mitad del siglo XII se diferencian claramente dos grupos de poetas segn su modo de expresin: los partidarios de una poesa sencilla el trobar leu y los que cultivan un arte ms complejo el trobar clus y el trobar ric.

El trobar leu
El trobar leu o arte de la versificacin sencilla fue practicado, entre otros, por el poeta Bernart de Ventadorn, que ha sido considerado el primero de los trovadores amorosos de la literatura provenzal. A l pertenece este fragmento prosificado.

Texto 1
Tengo mi corazn tan lleno de alegra que todo me lo transfigura: el fro me parece flor blanca, roja y amarilla, pues con el viento y con la lluvia me crece la ventura; por lo que mi mrito aumenta y sube y mi canto mejora. Tanto amor tengo en el corazn, tanta alegra y dulzura, que el hielo me parece flor y la nieve verdor.
Bernart de VENTADORN en Historia de la literatura universal, Planeta

El trobar clus y el trobar ric


Entre los poetas partidarios del arte de la poesa difcil, algunos practicaron el denominado trobar clus o cerrado, que utilizaba un lenguaje sumamente enigmtico, recargado de conceptos, aunque a veces incorporaba tambin un registro popular y bajo. Otro grupo de trovadores, por el contrario, se volcaron hacia el trobar ric, un arte ms complejo en el que se atenda esencialmente a la belleza formal, a la sonoridad, a la sugerencia del sonido y el ritmo, y a la seleccin de un vocabulario inusual. El trobar clus surgi entre poetas realistas y de condicin humilde, que rechazaban el amor corts, como Marcabr. El trobar ric, en cambio, posea un carcter aristocrtico, aceptaba los postulados de ese cdigo amoroso y no era en absoluto moralizante. Estos rasgos se observan, por ejemplo, en la obra de Arnaut Daniel.

Texto 2
Homicidas y traidores, simonacos1, saludadores, lujuriosos y usureros que viven de ruines menesteres, y los que hacen hechiceras y las repugnantes hechiceras estarn por igual en el fuego ardiente. Borrachos y cornudos, falsos curas y falsos abades, falsas reclusas y falsos reclusos, all penarn, dice Marcabr.
MARCABR

Texto 3
El aura amarga hace aclarar los bosquecillos ramosos, que la dulce espes con hojas, y mantiene balbucientes y mudos los alegres picos de los pjaros de las ramas, aparejados y no aparejados. Por qu yo me esfuerzo en hacer y decir cosas agradables a muchos? Por aquella que me ha vuelto de arriba abajo, de lo que temo morir si no me da fin a los afanes.
Arnaut DANIEL en Historia de la literatura universal Planeta

simonaco: el que compra o vende bienes espirituales.


1

1 Qu sentimiento embarga al yo potico del texto 1? Qu lo causa y qu efecto produce? 2 A quines ataca especialmente el poeta Marcabr en el fragmento seleccionado?

3 Qu anhela el emisor de la composicin de Arnaut Daniel? 4 Explica qu provoca en el emisor de texto 3 el estado en el que se encuentra.

MATERIAL FOTOCOPIABLE / Oxford University Press Espaa, S. A.

Evaluacin 10
Texto 1
Plega Dios que alguno quieras como yo, mi bien, te quiero, porque penes, porque mueras, porque sepas que de veras por tu sola causa muero. Amada ni ms ni menos seas como yo de ti, porque defectos ajenos te hagan mencin de m; y penes porque debieras quererme como te quiero; y por me cobrar te mueras y no puedas aunque quieras siendo yo muerto primero.

10

Pedro LVAREZ OSORIO CONDE DE TRASTMARA y MARQUS DE ASTORGA Poesa espaola 2. Edad Media: Lrica y cancioneros Crtica

Ilustracin de la Divina comedia, de Dante Alighieri.

Texto 2
Ojal alcanzara la unin mi corazn, que est en continua angustia! Todo corazn que arde en la pasin del amor est inquieto por la agitacin que sufre y que deshace al que en ansias se acongoja.
Ibn RUHAYM en F. CORRIENTE, Poesa dialectal rabe y romance en al-ndalus Gredos

Texto 4
A quin contar yo mis quejas, mi lindo amor? A quin contar yo mis quejas si a vos no?
Lrica espaola de tipo popular, Ctedra

Texto 5
Qu se hizo la moneda que guard para mis daos tantos tiempos, tantos aos, plata, joyas, oro o seda? Ca de todo no me queda sino este cadahalso. Mundo malo, mundo falso, no es quin contigo pueda.

Texto 3
Garid vos, ay yermaniellas!, cm contenir el mio male? Sin el habib non vivreyo: ad ob li rey demandare?
Lrica espaola de tipo popular Ctedra

(TRADUCCIN: Decidme, ay hermanitas!, / cmo contener mi mal? / Sin el amado no vivir: / adnde ir a buscarlo?)

Marqus DE SANTILLANA Poesa cancioneril castellana, Akal

1. El texto 1 corresponde a una cantiga de la poesa cancioneril del siglo xv. Reconoce en ella las caractersticas formales y temticas de este tipo de composiciones.

6. Los textos 3 y 4 brindan muestras de dos tipos de poesa


primitiva popular. Identifcalas y explica sus rasgos.

7. Qu tipo de poesa popular no aparece representada?


Seala semejanzas y diferencias con los anteriores.

2. El texto 2 es la traduccin prosificada de una estrofa de


una moaxaja rabe. Seala los rasgos propios de estos poemas y explica el contenido de la estrofa reproducida.

8. El texto 5 es una estrofa de una obra del marqus de


Santillana. Explica su contenido y analiza su mtrica. De qu tipo de poesa cancioneril es muestra?

3. Qu otra especie potica culta fue cultivada en alndalus? Explica en qu se diferencia de la moaxaja.

9. Cul es la gran creacin potica de Jorge Manrique?


Explica los siguientes aspectos de esa obra: Contenido de cada parte. Temas. Mtrica. Estilo.

4. Qu poesa se cultiv en el este de la Pennsula en los


siglos II y &III? Explica su vertiente amorosa.

5. En qu manifestacin culta de la poesa del oeste


peninsular influye la lrica aludida en la actividad 4? Comenta sus rasgos principales.

10. Redacta una exposicin sobre la poesa medieval culta


y popular en gallego-portugus.

ACTIVIDADE S

1. El Cantar de Mio Cid


La pica castellana tiene su mejor exponente en el Cantar de Mio Cid, que exalta la figura del infanzn Rodrigo Daz de Vivar, el Cid Campeador, tanto en su dimensin histrica como en las circunstancias personales de la vida del hroe.

MATERIAL FOTOCOPIABLE / Oxford University Press Espaa, S. A.

Comienzan ya a preparar la gran sala de palacio. Los suelos con mucha alfombra, todo bien encortinado. Cunta seda y cunta prpura y cunto pao preciado! Gusto os dara vivir y comer en el palacio! Los caballeros del Cid aprisa all se juntaron, y entonces en aquel punto por los Infantes mandaron. Ya cabalgan los Infantes, camino van del palacio, con muy ricas vestiduras, galanamente ataviados. A pie y con muy buena cara, Dios qu discretos entraron! Recibiolos nuestro Cid; con l todos sus vasallos. Ante el Cid y su mujer los Infantes se inclinaron. A sentar ellos se fueron en un muy precioso escao. Los de la casa del Cid, siempre en todo mesurados, estn atentos mirando al que naci afortunado. All el Cid Campeador ved que en pie se ha levantado: Puesto que hacerlo tenemos, por qu lo vamos tardando? Venid ac, mi Alvar Fez, el que tanto quiero y amo. Aqu tenis mis dos hijas; yo las pongo en vuestras manos. Sabis que al Rey esto mismo se lo tengo as rogado. No quiero faltar en nada de lo que fue concertado. A los dos Infantes, vos ddselas con vuestras manos, que tomen las bendiciones, y vayamos acabando. Entonces dijo Minaya: Esto har yo de buen grado. De pie las dos se levantan, y l las tom de las manos. Y a los Infantes, Minaya esto mismo les va hablando: Aqu estis ante el Minaya, vosotros, los dos hermanos. De manos del Rey Alfonso, pues a m me lo ha mandado, estas dueas yo os entrego, que ambas son hijasdalgo; que las tomis por mujeres, dadles honras y cuidados. Los de Carrin las reciben con amor y de buen grado. Al Cid y a doa Jimena, van a besarles las manos. As que esto hubieron hecho, se salieron del palacio. Aprisa, a Santa Mara, hacia all van caminando. Muy pronto se revisti don Jernimo, el prelado, y a la puerta de la iglesia estbalos esperando. Las bendiciones les dio; despus la misa ha cantado. Al salirse de la iglesia cabalgaron a buen paso. Afuera de la ciudad, en un arenal cercano, Dios, y qu buen juego de armas hizo el Cid a sus vasallos!

10

15

20

25

30

35

Poema del Cid Castalia

1 Qu episodio del Cantar narra este fragmento? Indica a qu parte del poema corresponde. 2 Seala las apelaciones al auditorio presentes en estos versos. Qu funciones del lenguaje predominan en ellos? 3 Reconoce el empleo de eptetos picos y seala a quin se califica por medio de este recurso. 4 Mide los versos y analiza la rima de esta tirada del Cantar de Mio Cid.

5 Diferencia la presencia del discurso del narrador del de los personajes en este fragmento. 6 Para relatar los sucesos, el narrador emplea dos tiempos verbales; de cules se trata? 7 Qu efecto se produce con el cambio de formas verbales? 8 Explica cmo se caracterizan los personajes mencionados en este pasaje.

ACTIVIDADE S

Un milagro, de Gonzalo de Berceo


A continuacin, tienes uno de los Milagros de Nuestra Seora, de Gonzalo de Berceo; lelo con atencin y observa su estructura, pues esta se repite en los restantes milagros que componen la coleccin.

El labrador avaro
rase en una tierra un hombre labrador que usaba de la reja ms que de otra labor; ms amaba la tierra que no a su Criador, y de muchas maneras era revolvedor. Haca una enemiga bien sucia de verdad: cambiaba los mojones por ganar heredad; haca en todas formas tuertos y falsedad, tena mal testimonio entre su vecindad. Aunque malo, quera bien a Santa Mara, oa sus milagros, muy bien los acoga; saludbala siempre, decale cada da: Ave grata plena que pariste al Mesas. Fin el arrastrapajas de tierra bien cargado, de los diablos fue luego en soga cautivado; lo arrastraban con cuerdas, de coces bien sobado, 15 le pechaban al doble el pan que dio mudado. Dolironse los ngeles de esta alma mezquina por cuanto la llevaban los diablos en rapina; quisieron socorrerla, ganarla por vecina, mas para hacer tal pasta mengubales harina. 20 Si les decan los ngeles de bien una razn, ciento decan los otros malas, que buenas non; los malos a los buenos tenan en un rincn, la alma por sus pecados no sala de prisin. 5 Levantndose, un ngel dijo: Yo soy testigo, verdad, es no mentira, esto que ahora os digo: el cuerpo que traa esta alma consigo fue de Santa Mara buen vasallo y amigo. Siempre la mencionaba al yantar y a la cena, decale tres palabras: Ave, grata plena. Boca por que sala tan santa cantilena no mereca yacer en tan mala cadena. Luego que este nombre de la Santa Rena oyeron los demonios, salieron tan ana, derramronse todos como una neblina, desampararon todos a esa alma mezquina. Los ngeles la vieron quedar desamparada, de manos y de pies con sogas bien atada, estaba como oveja cuando yace enzarzada: fueron y la llevaron junto con su majada. Nombre tan adornado, lleno de virtud tanta, y que a los enemigos los seguda y espanta, no nos debe doler ni lengua ni garganta que no digamos todos: Salve, Regina sancta.
Gonzalo DE BERCEO Milagros de Nuestra Seora Castalia

25

30

10

35

40

1 Caracteriza al protagonista de este milagro. Qu le sucede? 2 Resume por escrito el contenido de El labrador avaro. 3 Caracteriza al narrador de esta historia de Gonzalo de Berceo. 4 Diferencia en el milagro que acabas de leer las partes narrativas de las intervenciones de los personajes. 5 Cmo se introducen estas ltimas? 6 Seala las partes en que se divide la narracin de Berceo.

7 Reconoce los movimientos narrativos y los tipos de discurso que aparecen en el texto. 8 Cul es la finalidad de este milagro? Relacinala con la del resto de la obra a la que pertenece. 9 Se cumple en este milagro en concreto lo que en tu libro de texto se plantea en general para todas estas narraciones? Justifica tu respuesta. 10 Escribe en prosa la historia narrada por Berceo en El labrador avaro. No olvides cuidar la ortografa y la presentacin de tu redaccin.

MATERIAL FOTOCOPIABLE / Oxford University Press Espaa, S. A.

ACTIVIDADE S

10

Fragmentos del Libro de buen amor


Los dos fragmentos siguientes corresponden al Libro de buen amor. El primero, de carcter narrativo, pertenece a la serie de aventuras que el protagonista vive en la sierra, donde se enfrenta a mujeres de caractersticas muy especiales; el segundo, lrico, se inserta en el conjunto de poemas intercalados en la historia narrada. Estos dos textos, adems, presentan diferencias en cuanto a la mtrica empleada.

La serrana Aldara
Sus miembros y su talle no son para callar, me podis creer, era gran yegua caballar; quien con ella luchase mal se habra de hallar, si ella no quiere, nunca la podrn derribar. [] Tena la cabeza mucho grande y sin guisa cabellos cortos, negros, como corneja lisa, ojos hundidos, rojos; ve poco y mal divisa; mayor es que de osa su huella, cuando pisa. Las orejas, mayores que las de aal borrico, el su pescuezo, negro, ancho, velludo, chico; las narices, muy gordas, largas, de zarapico, sorbera bien pronto un caudal de hombre rico! Su boca era de alano, grandes labios muy gordos, dientes anchos y largos, caballunos, moxmordos; sus cejas eran anchas y ms negras que tordos. Los que quieran casarse, procuren no estar sordos! Mayores que las mas tiene sus negras barbas; yo no vi ms en ella, pero si ms escarbas, hallars, segn creo, lugar de bromas largas, aunque ms te valdr trillar en las tus parvas. [] Ms anchas que mi mano tiene la su mueca, velluda, pelos grandes y que nunca est seca; voz profunda y gangosa que al hombre da jaqueca, tarda, enronquecida, muy destemplada y hueca. Es su dedo meique mayor que mi pulgar, son los dedos mayores que puedes encontrar, que, si algn da, ella te quisiere espulgar, daarn tu cabeza cual vigas de lagar.
MATERIAL FOTOCOPIABLE / Oxford University Press Espaa, S. A.

Gozos de Santa Mara


Madre de Dios, gloriosa Virgen Santa Mara hija y leal esposa de tu hijo Mesa, 5 t, Seora, dame ahora la tu gracia a toda hora; que te sirva en toda va. Porque honrarte codicio, pecador yo!, por tanto ofrezco en tu servicio los gozos que te canto. El primero en que tercero a ti el ngel mensajero fue del Espritu Santo. Concebiste a tu Padre; fue tu gozo segundo, cuando pariste, Madre, sin dolor sali al mundo. 20 Cual naciste (t pariste), intacta permaneciste Oh Virgen del santo mundo! El tercero, la estrella, luciente meteoro; los Reyes, con luz de ella, trajeron su tesoro y alabaron y adoraron y a tu Hijo presentaron el incienso, mirra y oro. 25 5

10

10

15

15

20

25

30

Tena en el justillo las sus tetas colgadas, dbanle en la cintura porque estaban dobladas, que, de no estar sujetas, diranle en las ijadas; de la ctara al son bailan, aun no enseadas.

30

Arcipreste DE HITA Libro de buen amor Castalia

Arcipreste DE HITA Libro de buen amor Castalia

1 Atiende a la descripcin de la serrana Aldara y explica cmo aparece caracterizada. 2 Seala las referencias al receptor. Con qu finalidad aparecen? 3 Recuerda los rasgos de la pastorela y justifica por qu esta serrana es un texto pardico. 4 Explica los recursos que emplea el narrador para degradar la figura de la serrana.

5 Analiza la mtrica y la rima de los Gozos. 6 Seala la diferencia mtrica existente entre estos versos de los Gozos y la utilizada, en general, en el Libro de buen amor. 7 Con qu recursos se exalta en la segunda composicin la figura de la Virgen Mara? 8 Justifica por qu los Gozos de Santa Mara constituyen un texto de carcter lrico.

ACTIVIDADE S

13

Novela de caballeras y novela sentimental


Los siguientes fragmentos corresponden a dos de las principales orientaciones de la narrativa del siglo XV: la novela de caballeras y la novela sentimental.

Texto 1
Amads viva en una gran tristeza y soledad. Un da cabalgaba por la orilla del mar, y al subir a unas peas vio que se acercaba una nave. En la nave vena Durn, hermano de la doncella de Dinamarca. Gandaln lo abraz y lo condujo hasta donde estaba Amads. Cuando estuvieron cerca, vieron una especie de gigante envuelto en pieles que esgrima un venablo para arrojarlo contra Amads. Gandaln y Durn empezaron a dar grandes voces, y el venablo pas por encima de la cabeza de Amads. Amads se levant de un salto y pudo salvarse de un segundo venablo. Ech mano a la espada, pero aquella figura extraa sali huyendo, subi en el caballo de Amads y se alej, diciendo a grandes voces: Yo soy Andandona, la giganta de la Isla Triste, tu enemiga, y si ahora no acab lo que deseaba, otra vez ser. Amads dej que Gandaln la persiguiese y abraz a Durn y le pregunt por el objeto de su venida. Durn le dio una carta de Oriana y le dijo que ella haba tenido un hijo que criaba la abadesa del monasterio de Miraflores, y que le rogaba que no partiese de Gaula hasta tener su mando. Volvi Gandaln, con el caballo de Amads y la cabeza de Andandona atada al pretal por los cabellos largos y canos. Amads envi la cabeza a Bruneo de Bonamar y dijo a Durn: Vuelve y dile a tu seora que le beso las manos por la carta que me enva y por lo que t me has dicho de su parte, y que le ruego que tenga piedad de mi honra y no me deje holgar mucho tiempo, porque lo atribuiran a poquedad de corazn y perdera la gloria que por ella he ganado.
Amads de Gaula Castalia

Texto 2
Lucenda: es tan grande merced esta que hacerme quisiste, que si yo de ms valer o ella de menos sustancia no fuese, servicio ninguno satisfacerla podra, si pena en cuenta de servir recibida no fuese; la cual si recibir no quisieres, si merecida te la tengo en m lo puedes bien ver, porque en mi desfiguramiento a ella y a tus obras pintadas vers; y no solamente mis lgrimas a acrecentar mis dolores me han bastado, mas viendo mi parecer, a muchos enamorados de amar he hecho temerosos; y puesto que as es, te suplico que, arrepentida de lo pasado, lo por venir enmiendes. No seas en el daar siempre una; pon con tus obras mis guerras en paz; no s por qu pudiendo has dejado de servida ser. Todas lo quieren y t lo rehyes; bien parece que yo tengo mayor necesidad de tus mercedes, que t voluntad de mis servicios tenas. Oh qu combates de mi mucha fe y de tu poca esperanza he recibido!, los cuales, como ves, la fuerza de mi salud han enflaquecido. Cuanto t menos de mi dolor te dolas, ms mi dolor me dola. Si pudiese en la boca poner lo que en el alma he sentido, cunta culpa por mi pena te daras! Nunca nadie menos bien tuvo; nunca nadie ms mal sufri; nunca de tanta memoria tanto olvido se tuvo; mi afeccin y tu menosprecio destruyen mi salud. Todo esto, Lucenda, te digo, porque ms y en ms mi querer tengas, y porque en mi sufrimiento mi firmeza conozcas; que ni por todo mi mal jams en mudanza pens, ni de sufrir cansado me vi; antes ganado, porque t me perdas, siempre me hall, pero no por pequea causa, que no tena yo menos razn para penar que t hermosura para penarme.
MATERIAL FOTOCOPIABLE / Oxford University Press Espaa, S. A.

Diego DE SAN PEDRO Tratado de amores de Arnalte y Lucenda. Sermn Castalia

1 Describe cmo aparecen caracterizados los personajes del primer texto y justifica por qu son prototpicos del tipo de novela al que pertenecen. 2 En el primer pasaje se plantean dos situaciones; de cules se trata? A qu caractersticas de la novela de caballeras remite esta circunstancia? 3 Teniendo en cuenta lo que sabes acerca de la novela a la que pertenece el primer fragmento, explica la frase con la que concluye (perdera la gloria que por ella he ganado).

4 A qu clase de texto corresponde el fragmento de Diego de San Pedro? Explica de qu modelo narrativo del siglo XV es propio. 5 Justifica por qu su contenido tambin corresponde a ese tipo de novela. 6 Analiza en el fragmento de Diego de San Pedro recursos estilsticos de este tipo de textos. 7 Qu funciones del lenguaje predominan en cada uno de los fragmentos? 8 Compara los dos pasajes y seala las diferencias ms relevantes.

Evaluacin 10
Planto de Pleberio
PLEBERIO.Oh Fortuna variable, ministra y mayordoma de los temporales bienes! Por qu no ejecutaste tu cruel ira, tus mudables ondas, en aquello que a ti es sujeto? Por qu no destruiste mi patrimonio? Por qu no quemaste mi morada? Por qu no asolaste mis grandes heredamientos? Dejrasme aquella florida planta, en quien t poder no tenas. Dirasme, fortuna fluctuosa, triste la mocedad con vejez alegre; no pervirtieras el orden. Mejor sufriera persecuciones de tus engaos en la recia y robusta edad, que no en la flaca postrimera. Oh vida de congojas llena, de miserias acompaada! Oh mundo, mundo! Muchos mucho de ti dijeron, muchos en tus cualidades metieron la mano; a diversas cosas por odas te compararon. Yo por triste experiencia lo contar, como a quien las ventas y compras de tu engaosa feria no prsperamente sucedieron, como aquel que mucho ha hasta ahora callado tus falsas propiedades por no encender con odio tu ira, porque no me secases sin tiempo esta flor que este da echaste de tu poder. Pues ahora, sin temor, como quien no tiene qu perder, como aquel a quien tu compaa es ya enojosa, como caminante pobre que sin temor de los crueles salteadores va cantando en alta voz. Yo pensaba en mi ms tierna edad que eras y eran tus hechos regidos por algn orden; ahora, visto el pro y la contra de tus bienandanzas, me pareces un laberinto de errores, un desierto espantable, una morada de fieras, juego de hombres que andan en corro, laguna llena de cieno, regin llena de espinas, monte alto, campo pedregoso, prado

El suicidio de Melibea, grabado de la edicin de Valencia de 1514.

lleno de serpientes, huerto florido y sin fruto, fuente de cuidados, ro de lgrimas, mal de miserias, trabajo sin provecho, dulce ponzoa, vana esperanza, falsa alegra, verdadero dolor. Cbasnos, mundo falso, con el manjar de tus deleites; al mejor sabor nos descubres el anzuelo: no lo podemos huir, que nos tiene ya cazadas las voluntades. Prometes mucho, nada cumples. chasnos de ti, porque no te podamos pedir que mantengas vanos prometimientos. Corremos por los prados de tus viciosos vicios muy descuidados, a rienda suelta; descbresnos la celada cuando ya no hay lugar de volver.
Fernando DE ROJAS La Celestina, Crtica

1. Indica a qu parte de la historia de La Celestina corresponde este pasaje y reconoce en l recursos expresivos. Qu modalidad del discurso dramtico presenta este fragmento? Explica las caractersticas de las otras dos modalidades.

6. Compara el Cantar de Mio Cid con los poemas del mester


de clereca, considerando los aspectos indicados.

STER DE CLERECA

2. Qu relacin guarda la intervencin de Pleberio con el


tratamiento del amor en la obra? Comenta otros aspectos del amor considerados en la unidad.

7. Menciona obras del mester de clereca que estn escritas


en cuaderna va.

8. Qu son los enxiemplos ? Indica cmo se insertan en


El conde Lucanor.

3. Expn cuanto sepas acerca del gnero y los personajes


principales de La Celestina.

9. Explica la estructura y el sentido del Libro de buen amor.


Qu personaje incorpora esta obra medieval a la tradicin literaria?

4. En cierta forma, la obra de Fernando de Rojas parodia la


novela sentimental. Explica por qu y compara este tipo de ficcin narrativa del siglo xv con la de caballeras, atendiendo a los aspectos que se indican en la tabla.

10. Redacta una exposicin acerca de los romances que


contemple los aspectos sealados a continuacin. Caractersticas de los romances viejos. Forma de transmisin de dichas composiciones. Temas, estructura narrativa y recursos expresivos caractersticos. 215 13.
La narrativa medieval

CABALLERAS
Cantar de Mio Cid.

SENTIMENTAL

. asuntos que determinan la estructura del 5. Explica los dos

ACTIVIDADE S

11

Obra potica de Garcilaso de la Vega


En la obra potica de Garcilaso, el amor es el tema ms importante, con el que se relacionan los de la naturaleza y la mitologa. Estos temas estn presentes en sus sonetos y en las glogas.

Soneto XXIX
Pasando el mar Leandro el animoso, en amoroso fuego todo ardiendo, esforz el viento1 y fuese embraveciendo el agua con un mpetu furioso. Vencido del trabajo presuroso, contrastar2 a las ondas no pudiendo, y ms del bien que all perda muriendo que de su propia vida congojoso3, como pudo, sforz su voz cansada y a las ondas habl desta manera, mas nunca fue su voz dellas oda: Ondas, pues no se escusa que yo muera, dejadme all llegar, y a la tornada4 vuestro furor esecut5 en mi vida. 5

gloga II
ALBANIO: En aquel prado all nos reclinamos, 13 y, del Cfiro fresco recogiendo el agradable espirtu14, respiramos; las flores, a los ojos ofreciendo diversidad estraa de pintura15, diversamente as estaban oliendo; y en medio aquesta fuente clara y pura, que como de cristal resplandeca, mostrando abiertamente su hondura, el arena, que doro pareca, de blancas pedrezuelas varada, por do manaba el agua, se bulla. 10 5

10

esforz el viento: se desat el vendaval. contrastar: no pudiendo resistir las olas. congojoso: preocupado. tornada: vuelta. d: dnde agora: ahora. derrota. esecut: ejecutad.

gloga I
NEMOROSO: D6 estn agora7 aquellos claros ojos que llevaban tras s, como colgada, mi alma, doquier8 que ellos se volvan? Do est la blanca mano delicada, llena de vencimientos9 y despojos, que de m mis sentidos l`ofrecan? Los cabellos que van con gran desprecio al oro como a menor tesoro adnde estn, adnde el blanco pecho? 10 D la columna10 quel dorado techo11 con proporcin graciosa sostena? Aquesto todo agora ya sencierra, por desventura ma, en la escura12, desierta y dura tierra.

2 3

4 5

7 8 9

doquier: donde quiera vencimiento: columna: el cuello. dorado techo: la rubia cabellera. escura: oscura. espirtu: aire.

10 11 12 13

MATERIAL FOTOCOPIABLE / Oxford University Press Espaa, S. A.

Cfiro: viento suave y apacible. estraa de pintura: diversidad extraordinaria de colores.

14

15

15

1 Lee el Soneto XXIX y explica qu le ocurre a Leandro. Qu sentimientos le impulsan a esforzarse? 2 A quines se dirige en los tercetos y qu les pide? 3 Qu descripcin de la amada realiza Nemoroso en la Egloga I ? 4 A qu imagen corresponden los rasgos que has sealado en la pregunta anterior?

5 Qu tpico recrean las interrogaciones que estructuran el fragmento de la gloga I? 6 Qu rasgos tiene el paisaje descrito en la gloga II? A qu tpico responde? 7 Analiza la mtrica y los principales recursos expresivos de las tres composiciones. 8 Relaciona los temas y tpicos de los textos con los de la poesa renacentista.

ACTIVIDADE S

11

Fray Luis de Len


Fray Luis sufri un proceso inquisitorial que lo llev a la crcel. El agustino se sinti acosado y crey que la envidia de otros, sobre todo dominicos, haba sido determinante; a esto se refiere en el primer poema reproducido. La segunda composicin forma parte de sus poemas religiosos.

A la salida de la crcel
Aqu la envidia y mentira me tuvieron encerrado. Dichoso el humilde estado del sabio que se retira de aqueste mundo malvado, y con pobre mesa y casa en el campo deleitoso con solo Dios se compasa y a solas su vida pasa, ni envidiado ni envidioso.

En la Ascensin
Y dejas, Pastor santo, tu grey1 en este valle hondo, escuro, con soledad y llanto; y t, rompiendo el puro aire, te vas al inmortal seguro2? Lo antes bienhadados y los agora tristes y afligidos, a tus pechos criados, de ti desposedos, a d convertirn3 ya sus sentidos? Qu mirarn los ojos, que vieron de tu rostro la hermosura, que no les sea enojos? Quien oy tu dulzura, qu no tendr por sordo4 y desventura? Aqueste mar turbado quin le pondr ya freno?, quin concierto al viento fiero, airado?; estando t encubierto, qu norte guiar la nave al puerto? Ay nube envidiosa!: aun deste breve gozo, qu te aquejas? D vuelas presurosa? Cun rica t te alejas! Cun pobres y cun ciegos, ay, nos dejas!
1 2 3

20

10

25

convertirn: volvern, dirigirn.

15

sordo: sonido desprovisto de armona.

1 Adems del tema de la envidia, qu otros temas desarrolla el primer poema? 2 Qu tpico aparece? Relaciona este texto con la Oda a la vida retirada (pgina 227 de tu libro de texto). 3 Qu suceso religioso se canta en el segundo poema? 4 Explica a quin se dirige en la primera estrofa y qu le reprocha.

5 Cules sern los sentimientos de los apstoles, despus de la partida de Jess? 6 A qu pueden referirse las metforas del mar, el viento fiero, la nave y el puerto de la siguiente estrofa? 7 La nube envidiosa tapa la subida de Cristo, cmo se quedan los apstoles? 8 Realiza el anlisis mtrico del primer poema. Cul es la estrofa utilizada en el segundo?

MATERIAL FOTOCOPIABLE / Oxford University Press Espaa, S. A.

grey: conjunto de fieles cristianos. seguro: lugar desprovisto de todo peligro.

ACTIVIDADE S

11

3. San Juan de la Cruz


El Cntico espiritual es una de las obras maestras de la poesa mstica. En l, san Juan utiliza el lenguaje simblico para referirse a la bsqueda que realiza la Esposa (el alma) de su amado (Dios), junto con la valoracin de la naturaleza y numerosos recursos expresivos.

Cntico espiritual
ESPOSA: Adnde te escondiste, Amado, y me dejaste con gemido? Como el ciervo huiste, habindome herido; sal tras ti clamando, y eras ido. Pastores, los que fuerdes all por las majadas al otero2, si por ventura vierdes aquel que yo ms quiero, decildle que adolezco3, peno y muero. Buscando mis amores ir por esos montes y riberas; ni coger las flores, ni temer las fieras, y pasar los fuertes y fronteras. (PREGUNTA A LAS CRIATURAS) Oh, bosques y espesuras, plantadas por la mano del Amado! Oh, prado de verduras, de flores esmaltado!, decid si por vosotros ha pasado RESPUESTA DE LAS CRIATURAS: Mil gracias derramando, pas por estos sotos con presura4, e, yndolos mirando, con sola su figura vestidos los dej de hermosura. ESPOSA: Ay, quin podr sanarme! Acaba de entregarte ya de vero5; no quieras enviarme de hoy ms ya mensajero, que no saben decirme lo que quiero. []
1

ESPOSA: Gocmonos, Amado, y vmonos a ver en tu hermosura al monte o al collado, do mana el agua pura; entremos ms adentro en la espesura. Y luego a las subidas cavernas de la piedra nos iremos, que estn bien escondidas, y all nos entraremos y el mosto de granadas gustaremos. All me mostraras aquello que mi alma pretenda, y luego me daras all, t, vida ma, aquello que me diste el otro da: el aspirar del aire, el canto de la dulce filomena7, el soto y su donaire en la noche serena, con llama que consume y no da pena. 20
6

35

10

40

15

45

50

San Juan DE LA CRUZ Cntico espiritual, Crtica

fuerdes / vierdes: formas arcaicas de fueres y vieres. otero: cerro aislado que domina un llano. adolecer: enfermar.

25
4 5 6

presura: prisa, prontitud, ligereza. de vero: de verdad. aspirar: el soplo. filomena: el ruiseor.

MATERIAL FOTOCOPIABLE / Oxford University Press Espaa, S. A.

30

1 De qu se queja la amada? Cmo se siente? 2 Qu les pide a los pastores? Qu decide hacer, segn la tercera estrofa y a qu est dispuesta? 3 La amada se dirige a la naturaleza, cmo se presenta sta? Qu le responde? 4 Qu le reprocha al amado en la sexta estrofa? Qu rechaza?

5 En los versos 31-35 ya se ha producido la unin, explica cul es el ruego de la esposa, a dnde le sugiere que vayan? 6 Qu desea la esposa en la ltima estrofa? 7 Analiza la mtrica del poema. 8 Localiza y explica los principales recursos expresivos empleados por san Juan en estos versos.

ACTIVIDADE S

11

4. Lrica renacentista
En esta pgina se incluyen poemas de otros autores renacentistas, que siguieron el modelo de Garcilaso de la Vega: Gutierre de Cetina (1510-1554), Fernando de Herrera (1534-1597), Francisco de Aldana (1537-1578). Del bellsimo ltimo texto, annimo, no se sabe exactamente su fecha.

Texto 1
Entre armas, guerra, fuego, ira y furores, que al soberbio francs tienen opreso1, cuando el aire es ms turbio y ms espeso, all me aprieta el fiero ardor de amores. Miro el cielo, los rboles, las flores, y en ellos hallo mi dolor expreso; que en el tiempo ms fro y ms avieso2 nacen y reverdecen mis temores. Digo llorando: Oh dulce primavera!, cundo ser que a mi esperanza vea, verde, prestar al alma algn sosiego? Mas temo que mi fin mi suerte fiera tan lejos de mi bien quiere que sea entre guerra y furor, ira, armas, fuego.
Gutierre DE CETINA
1

Texto 3
De sus hermosos ojos dulcemente un tierno llanto Filis despeda que por el rostro amado pareca claro y precioso aljfar5 transparente; 5 en brazos de Damn, con baja frente, triste, rendida, muerta, helada y fra, estas palabras breves le deca, creciendo a su llorar nueva corriente: Oh pecho duro, oh alma dura y llena de mil durezas! Dnde vas huyendo? D vas con ala tan ligera y presta? Y l, soltando de llanto amarga vena, della las dulces lgrimas bebiendo, besola, y solo un ay! fue su respuesta.
Francisco DE ALDANA
5

10

10

opreso: oprimido.

avieso: malo, torcido.

aljfar: perla pequea de forma irregular.

Texto 2
Serena Luz, en quien presente espira divino amor, que enciende y junto enfrena4 el noble pecho, que en mortal cadena al alto Olimpo levantarse aspira; ricos cercos dorados, do se mira tesoro celestial de eterna vena; , armona de anglica sirena, que entre las perlas y el coral respira, cul nueva maravilla, cul ejemplo de la inmortal grandeza nos descubre aquesta sombra del hermoso velo? Que yo en esa belleza que contemplo (aunque a mi flaca vista ofende y cubre), la inmensa busco, y voy siguiendo al cielo.
Fernando DE HERRERA
3 3

Texto 4 Soneto a Cristo crucificado


No me mueve, mi Dios, para quererte el cielo que me tienes prometido; ni me mueve el infierno tan temido para dejar por eso de ofenderte. T me mueves, seor; muveme el verte 5 clavado en una cruz y escarnecido; muveme ver tu cuerpo tan herido; muvenme tus afrentas y tu muerte.
MATERIAL FOTOCOPIABLE / Oxford University Press Espaa, S. A.

10

Muveme, en fin, tu amor, y en tal manera que aunque no hubiera cielo, yo te amara, y aunque no hubiera infierno, te temiera. No tienes que me dar porque te quiera, pues aunque cuanto espero no esperara, lo mismo que te quiero te quisiera.

10

espira: sopla.

enfrena: frena.

ANNIMO Poesa lrica del Siglo de Oro, Ctedra

1 En qu situacin se halla el yo potico en el texto 1? Cules son sus sentimientos? 2 Cules son sus deseos y sus temores, segn los tercetos? Qu temas desarrolla este texto? 3 Qu descripcin se hace de la amada en el texto 2? 4 Qu visin de ella nos ofrece la voz potica? Relaciona esta imagen con las ideas del neoplatonismo.

5 En el texto 3, hay una escena de amor correspondido; sin embargo muestra una situacin dolorosa, cul es? 6 Cul es la reaccin y la respuesta del amante? 7 Explica las razones del amor a Cristo del yo potico en el ltimo texto. 8 Relaciona los temas de estos poemas con los de la lrica renacentista.

Evaluacin 11

Concierto campestre, por Tiziano.

El lamento de Albanio
En medio del invierno est templada el agua dulce desta clara fuente, y en el verano ms que nieve helada. Oh claras ondas, cmo veo presente, en vindoos, la memoria da quel da de que el alma temblar y arder se siente! En vuestra claridad vi mi alegra oscurecerse toda y enturbiarse; cuando os cobr, perd mi compaa. A quin pudiera igual tormento darse, que con lo que descansa otro afligido venga mi corazn a atormentarse? El dulce murmurar deste rido, el mover de los rboles al viento, el suave olor del prado florecido podrian tornar d enfermo y descontento cualquier pastor del mundo alegre y sano: yo solo en tanto bien morir me siento. Oh hermosura sobre l ser humano, oh claros ojos, oh cabellos d oro, oh cuello de marfil, oh blanca mano! Cmo puede ora ser que n triste lloro se convirtiese tan alegre vida y en tal pobreza todo mi tesoro? Quiero mudar lugar y a la partida quiz me dejar parte del dao que tiene el alma casi consumida.

1. Explica el contenido de este fragmento de la gloga II,


de Garcilaso de la Vega.

2. Menciona el tema del poema y reconoce los tpicos que


aparecen. 5

3. Justifica el carcter literario y el gnero de este texto. 4. Seala los principales recursos expresivos y realiza el
anlisis mtrico de la composicin.

5. Atendiendo a tus respuestas, relaciona el tema y los


10 tpicos, los recursos expresivos y la mtrica de estos versos con los principales rasgos de la lrica renacentista.

6. El fragmento citado pertenece a la gloga II. Explica qu es


una gloga y comenta el contenido de las tres que escribi Garcilaso de la Vega. Menciona tambin los temas principales de su produccin potica.

15

7. Qu innovaciones de la lrica renacentista proceden de


Italia? Por qu se dio esta influencia?

8. Qu tipos de poesa religiosa se cultivaron en la segunda


20 mitad del siglo & VI? Cita los ttulos de los poemas mayores de san Juan de la Cruz, explica el asunto principal abordado en su poesa y el recurso bsico que utiliza.

9. Cules son los temas y tpicos ms importantes de la


Oda a la vida retirada, de fray Luis de Len? Explica qu mtrica utiliza en la mayora de sus poemas. 25

10. Redacta un texto con las etapas y las tendencias poticas


de la poesa del siglo & VI. Cita autores y obras.

ACTIVIDADE S

12

1. Novela bizantina y novela pastoril


Los amores de Clareo y Florisea y los trabajos de la sin ventura Isea, de Alonso Nez de Reinoso, y La Diana, de Jorge de Montemayor, son obras paradigmticas de la novela bizantina y de la novela pastoril, respectivamente, en la pennsula.

En la fuente de los alisos


Despus que Sireno puso fin a su canto, vio como hacia l vena la hermosa Selvagia, y el pastor Sylvano, de que no recibi pequeo contentamiento, y despus de haberse recibido, determinaron ir a la fuente de los alisos, donde el da antes haban estado. Y primero que all llegasen, dijo Sylvano: Escucha, Selvagia, no oyes cantar? S oigo dijo Selvagia, y aun parece ms de una voz. A dnde ser? dijo Sireno. Parceme respondi Selvagia que es en el prado de los laureles, por donde pasa el arroyo que corre de esta clara fuente. Bien ser que nos lleguemos all, y de manera que no nos sientan los que cantan, porque no interrumpamos la msica. Vamos dijo Selvagia. Y as su paso a paso se fueron hacia aquella parte donde las voces se oan, y escondindose entre unos rboles que estaban junto al arroyo, vieron sobre las doradas flores asentadas tres ninfas, tan hermosas que pareca haber en ellas dado la naturaleza muy clara muestra de lo que puede. Venan vestidas de unas ropas blancas, labradas por encima de follajes de oro, sus cabellos que los rayos del sol oscurecan, revueltos a la cabeza, y tomados con sendos hilos de orientales perlas, con que encima de la cristalina frente se haca una lazada, y en medio della estaba un guila de oro, que entre las uas tena un muy hermoso diamante.
Jorge DE MONTEMAYOR La Diana, Ctedra

La nsula de la crueldad
Pasadas todas estas cosas, yo determin de partirme en Efeso; y mandando a fletar una nao y despidindome de Ibrina y de su marido, nos embarcamos y comenzamos de navegar la va de la ciudad de Efeso. Y habiendo algunos das que con buen tiempo hacamos nuestro camino, vimos de lejos una isla, la cual pareca tan oscura que la noche no lo es tanto. Pareca que unos humos negros de azufre salan della; las casas y arboledas eran todas negras y de negra color, las aguas que por ella corran eran todas de color de sangre; oanse grandes y dolorosos gritos y grandes alaridos que ponan espanto a los que los oan. Y ans, nosotros quedamos espantados y con deseo de saber qu aventura fuese aquella. Y porque ya era de noche, mandamos a los marineros que detuviesen la nao hasta la maana porque queramos saber qu tierra fuese aquella. Ellos lo hiceron ans y dijeron que era mejor tomar tierra porque aquella, aunque tan triste pareciese, era segura, a la cual se llamaba la nsula de la Crueldad, porque en ella estaban sepultados todos aquellos que cruelmente haban sido muertos, y que ans era el uso de aquellas tierras que estaban cercanas y de ms lejos, que era traer all a sepultallos. Yo quisiera que pasramos adelante por no quedar all en tan triste tierra, pero Clareo no lo consinti, antes mand tomar tierra. Y ans lo hicimos; y saliendo al puerto nos quedamos all la noche, la cual fue tan larga que pensamos que nunca amaneciese, y era la causa porque el sol no pareca en aquella isla, antes hua della. Pero habiendo entrado el da, aunque oscuro, bien conocimos que era ya de da, y comenzando a mirar la tierra vimos muchas sepulturas de muchas personas que cruelmente haban sido muertas.
Alonso NEZ DE REINOSO Los amores de Clareo y Florisea y los trabajos de la sin ventura Isea, SPUE

MATERIAL FOTOCOPIABLE / Oxford University Press Espaa, S. A.

1 Resume el contenido de ambos fragmentos. 2 Explica por qu constituyen textos narrativos. 3 Reconoce en los dos pasajes: Narrador. Movimientos narrativos. Tipos de discurso.

4 Seala los recursos expresivos con los que se materializa en cada uno de los fragmentos la funcin potica. 5 Indica los rasgos por los que los textos reproducidos en esta pgina se adscriben a la novela pastoril, el primero, y a la bizantina, el segundo.

ACTIVIDADE S

12

Novela celestinesca y novela morisca


A continuacin presentamos un par de textos representativos de dos de los modelos narrativos cultivados en el siglo XVI: la novela celestinesca y la novela morisca. Se trata, respectivamente, de Segunda Celestina, de Feliciano de Silva, y de El Abencerraje, que apareci incorporado en una edicin de La Diana.

La victoria de Narvez
Rodrigo de Narvez, barruntando la necesidad en que sus compaeros estaban, atraves el camino, y como traa mejor caballo se adelant; y viendo la valenta del moro, qued espantado, porque de los cinco escuderos tena los cuatro en el suelo, y el otro, casi al mismo punto. l le dijo: Moro, vente a m, y si t me vences, yo te aseguro de lo dems. Y comenzaron a trabar brava escaramuza, mas como el alcaide vena de refresco, y el moro y su caballo estaban heridos, dbale tanta priesa que no poda mantenerse; mas viendo que en sola esta batalla le iba la vida y contentamiento, dio una lanzada a Rodrigo de Narvez que, a no tomar el golpe en su darga, le hubiera muerto. l, en recibiendo el golpe, arremeti a l y diole una herida en el brazo derecho, y cerrando luego con l, le trab a brazos y, sacndole de la silla, dio con l en el suelo. Y yendo sobre l le dijo: Caballero, date por vencido; si no, matarte he. Matarme bien podrs dijo el moro que en tu poder me tienes, mas no podr vencerme sino quien una vez me venci. El alcaide no par en el misterio con que se decan estas palabras, y usando en aquel punto de su acostumbrada virtud, le ayud a levantar, porque de la herida que le dio el escudero en el muslo y de la del brazo, aunque no eran grandes, y del gran cansancio y cada, qued quebrantado; y tomando de los escuderos aparejo, le lig las heridas.
El Abencerraje (novela y romancero) Ctedra

1 Resume el contenido de ambos fragmentos. 2 Explica la peculiaridad del discurso del primer texto. 3 Reconoce en el segundo texto los siguientes aspectos: Narrador. Movimiento narrativo. Tipos de discurso.

4 Seala los principales recursos expresivos con los que se materializa en los fragmentos de Segunda Celestina y del Abencerraje la funcin potica. 5 Indica los rasgos por los que estos textos pertenecen a la novela celestinesca, el primero, y a la morisca, el segundo. No olvides argumentar tu respuesta.

MATERIAL FOTOCOPIABLE / Oxford University Press Espaa, S. A.

ACTIVIDADE S

12

Lazarillo de Tormes
El siguiente fragmento corresponde al Tratado III, que pertenece al primer mdulo de la obra, donde se cuenta la infancia de Lzaro de Tormes.

Al servicio del escudero


La maana venida, levantmonos, y comienza a limpiar y sacudir sus calzas1 y jubn2 y sayo3 y capa; y yo que le serva de pelillo4. Y vsteseme muy a su placer, despacio. Echele aguamanos5, peinose y psose su espada en el talabarte6 y, al tiempo que la pona, djome: Oh, si supieses, mozo, qu pieza es esta! No hay marco de oro en el mundo por que yo la diese. Mas ans ninguna de cuantas Antonio hizo no acert a ponelle7 los aceros tan prestos como esta los tiene. Y sacola de la vaina y tentola con los dedos, diciendo: Vesla aqu? Yo me obligo con ella cercenar un copo de lana. Y yo dije entre m: Y yo con mis dientes, aunque no son de acero, un pan de cuatro libras. Tornola a meter y cisela, y un sartal8 de cuentas gruesas del talabarte. Y con un paso sosegado y el cuerpo derecho, haciendo con l y con la cabeza muy gentiles meneos, echando el cabo de la capa sobre el hombro y a veces so el brazo, y poniendo la mano derecha en el costado, sali por la puerta, diciendo: Lzaro, mira por la casa en tanto que yo voy a or misa, y haz la cama y ve por la vasija de agua al ro, que aqu bajo est, y cierra la puerta con llave, no nos hurten algo, y ponla aqu al quicio, porque si yo viniere en tanto pueda entrar. Y sbese por la calle arriba con tal gentil semblante y continente, que quien no lo conociera pensara ser muy cercano pariente al conde de Arcos, o al menos camarero que le daba de vestir. Bendito seis Vos, Seor qued yo diciendo, que dais la enfermedad y ponis el remedio! Quin encontrar a aquel mi seor que no piense, segn l contento de s lleva, haber anoche bien cenado y bien dormido en buena cama, y, aun agora es de maana, no le cuenten por muy bien almorzado? Grandes secretos son, Seor, los que Vos hacis y las gentes ignoran! A quin no empear aquella buena disposicin y razonable capa y sayo? Y quin pensar que aquel gentil hombre se pas todo el da sin comer, con aquel mendrugo de pan que su criado Lzaro trujo9 un da y una noche en el arca de su seno, do no se le poda pegar mucha limpieza, y hoy, lavndose las manos y cara, a falta de pao de manos se haca servir de la halda10 del sayo? Nadie, por cierto, lo sospechar. Oh, Seor, y cuntos de aquestos debis Vos tener por el mundo derramados, que padecen por la negra que llaman honra lo que por Vos no sufriran!
Lazarillo de Tormes Ctedra

MATERIAL FOTOCOPIABLE / Oxford University Press Espaa, S. A.

calza: prenda que cubra, cindolos, el muslo y la pierna. 2 jubn: vestidura que cubra desde los hombros hasta la cintura, ceida y ajustada al cuerpo. 3 sayo: prenda de vestir holgada y sin botones que cubra el cuerpo hasta las rodillas. 4 servir de pelillo: hacer servicios de poca importancia.
1

aguamanos: aguamanil; agua que sirve para lavar las manos. 6 talabarte: cinturn de cuero del que cuelga la espada. 7 ponelle: ponerle. 8 sartal: sarta de cosas metidas en un hilo o cuerda. 9 trujo: trajo (de traer). 10 halda: falda.

1 Resume el contenido de este fragmento, correspondiente al Tratado III del Lazarillo y caracteriza el comportamiento del escudero. 2 En el primer prrafo se observa un cambio en el uso de los tiempos verbales. Qu efecto produce? Este recurso se reitera ms adelante; localzalo.

3 Qu movimiento narrativo se aprecia en la primera parte del fragmento? Por qu tipo de discurso es interrumpido? 4 Indica a qu tipo de discurso pertenece la exclamacin final. 5 Seala y explica el empleo del recurso de la irona en las palabras del escudero.

ACTIVIDADE S

12

Don Quijote de la Mancha


El siguiente texto pertenece a la segunda parte del Quijote. Los protagonistas han llegado al palacio de los duques, quienes, habiendo ledo la primera parte del libro, deciden invitarlos para divertirse a su costa. La duquesa le pide al caballero que describa a Dulcinea, y este comienza su respuesta aludiendo al encantamiento de que ha sido objeto (recuerda el engao de Sancho).

El encantamiento de Dulcinea
S hiciera, por cierto respondi don Quijote, si no me la hubiera borra do de la idea la desgracia que poco ha que le sucedi, que es tal, que ms estoy para llorarla que para describirla, porque habrn de saber vuestras grandezas que yendo los das pasados a besarle las manos, y a recebir su bendicin, beneplcito y licencia para esta tercera salida, hall otra de la que buscaba: hallela encantada y convertida de princesa en labradora, de hermosa en fea, de ngel en diablo, de ol orosa en pestfera, de bien hablada en rstica, de repo sada en brincadora, de luz en tinieblas, y, finalmen te, de Dulcinea del Toboso en una villana de Sayago. Vlame Dios! dando una gran voz, d ijo a este instante el duque. Quin ha sido el que tanto mal h a hecho al mundo? Quin ha quitado dl la belleza que le al egraba, el donaire que le entretena y la honestidad que le acreditaba? Quin? respondi don Quijote. Quin puede ser sino algn maligno encantador de los muchos invidiosos que me persiguen? Esta raza maldita, nacida en el mundo para escurecer y aniquilar las hazaas de los buenos, y para dar luz y levantar los fechos de los malos. Perseguido me han encantadores, encantadores me persiguen, y encantadores me perseguirn hasta dar conmigo y con mil altas caballeras en el profundo abismo del olvido, y en aquella parte que me daan y hieren donde ven que ms lo siento, porque quitarle a un caballero andante su dama es quitarle los ojos con que mira, y el sol con que se alumbra, y el sustento con que se mantiene. Otras muchas veces lo he dicho, y ahora lo vuelvo a decir; que el caballero andante sin dama es como el rbol sin hojas, el edificio sin cimiento, y la sombra sin cuerpo de quien se cause. No hay ms que decir dijo la duquesa; pero si, con todo eso, hemos de dar crdito a la historia que del seor don Quijote de pocos das a esta parte ha salido a la luz del mundo, con aplauso de las gentes, della se colige, si mal no me acuerdo, que nunca vuesa merced ha visto a la seora Dulcinea.

Y que esta tal seora no es en el mundo, sino que es dama fantstica, que vuesa merced la engendr y pari en su entendimiento, y la pint con todas aquellas gracias y perfecciones que quiso. En eso hay mucho que decir respondi don Quijote. Dios sabe si hay Dulcinea o no en el mundo, o si es fantstica, o no es fantstica; y estas no son de las cosas cuya averiguacin se ha de llevar hasta el cabo. Ni yo engendr ni par a mi seora, puesto que la contemplo como conviene que sea una dama que contenga en s las partes que puedan hacerla famosa en todas las del mundo, como son: hermosa sin tacha, grave sin soberbia, amorosa con honestidad, agradecida por corts, corts por bien criada y, finalmente, alta por linaje, a causa que sobre la buena sangre resplandece y campea la hermosura con ms grados de perfeccin que en las hermosas humildemente nacidas.
Miguel DE CERVANTES Don Quijote de la Mancha Planeta

1 Comenta los temas que se desarrollan en el dilogo entre don Quijote y los duques y relacinalos con las caractersticas del personaje protagonista y con lo que conoces del argumento de la novela. 2 Repasa las principales caractersticas de las novelas de caballeras y del concepto del amor corts y relacinalas con lo que expresa el caballero.

3 Qu responde el protagonista respecto al comentario de que Dulcinea es una dama fantstica? 4 Explica las caractersticas de la locura de don Quijote. 5 Analiza los recursos expresivos utilizados en la primera y la ltima intervencin de don Quijote en el fragmento que acabas de leer.

MATERIAL FOTOCOPIABLE / Oxford University Press Espaa, S. A.

ACTIVIDADE S

12

Una imagen de mujer en el Quijote


A continuacin se presentan dos textos del Quijote. El primero corresponde al discurso que pronuncia el caballero sobre la Edad de Oro. El segundo fragmento pertenece a la historia pastoril intercalada de Marcela y Grisstomo. Ambos jvenes hermosos y ricos campesinosdeciden hacerse pastores: Marcela porque prefiere este tipo de vida; Grisstomo porque pretende conquistarla. Marcela lo rechaza, pues no desea casarse, y Grisstomo se quita la vida. En este texto, la pastora hace su alegato.

Texto 1
Dichosa edad y siglos dichosos aquellos a quien los antiguos pusieron nombre de dorados, y no porque en ellos el oro, que en esta nuestra edad de hierro tanto se estima, se alcanzase en aquella venturosa sin fatiga alguna, sino porque entonces los que en ella vivan ignoraban estas dos palabras de tuyo y mo []. Las doncellas y la honestidad andaban, como tengo dicho, por dondequiera, sola y seora, sin temor que la ajena desenvoltura y lascivo intento la menoscabasen, y su perdicin naca de su gusto y propia voluntad.
Miguel DE CERVANTES Don Quijote de la Mancha Planeta

Texto 2
Yo nac libre, y para poder vivir libre escog la soledad de los campos. Los rboles de estas montaas son mi compaa, las claras aguas de estos arroyos mis espejos; con los rboles y con las aguas comunico mis pensamientos y hermosura. Fuego soy apartado y espada puesta lejos. A los que he enamorado con la vista he desengaado con las palabras. Y si los deseos se sustentan con esperanzas, no habiendo yo dado alguna a Grisstomo ni a otro alguno, en fin de ninguno de ellos, bien se puede decir que antes le mat su porfa que mi crueldad. Y si se me hace cargo que eran honestos sus pensamientos, y que por esto estaba obligada a corresponder a ellos, digo que cuando en este mismo lugar donde ahora se cava su sepultura me descubri la bondad de su intencin, le dije yo que la ma era vivir en perpetua soledad, y que sola la tierra gozase el fruto de mi recogimiento y los despojos de mi hermosura; y si l, con todo este desengao, quiso porfiar contra la esperanza y navegar contra el viento, qu mucho que se anegase en la mitad del golfo de su desatino? [] Si yo conservo mi limpieza con la compaa de los rboles, por qu ha de querer que la pierda el que quiere que la tenga con los hombres? Yo, como sabis, tengo riquezas propias y no codicio las ajenas; tengo libre condicin y no gusto de sujetarme; ni quiero ni aborrezco a nadie. No engao a este, ni solicito a aquel; ni burlo con uno, ni me entretengo con el otro. La conversacin honesta de las zagalas destas aldeas y el cuidado de mis cabras me entretiene. Tienen mis deseos por trmino estas montaas, y si de aqu salen, es a contemplar la hermosura del cielo, pasos con que camina el alma a su morada primera.
Miguel DE CERVANTES Don Quijote de la Mancha Planeta

MATERIAL FOTOCOPIABLE / Oxford University Press Espaa, S. A.

1 Redactad en grupos un informe en el que se relacionen los siguientes aspectos. Los pensamientos de don Quijote sobre la Edad de Oro. El alegato de Marcela.

Escribe un texto en el que reflexiones acerca de la vinculacin de la imagen de la mujer independiente planteada en estos textos con las concepciones de la mujer vigentes en la sociedad actual.

3 Analiza los principales recursos expresivos utilizados en el texto 2.

Evaluacin 12
Encuentro con el escudero
- T, mozo, has comido? - No, seor - dije yo- , que an no eran dadas las ocho cuando con Vuestra Merced encontr. - Pues, aunque de maana, yo haba almorzado, y cuando ans como algo, hgote saber que hasta la noche me estoy ans. Por eso, psate como pudieres, que despus cenaremos. Vuestra Merced crea, cuando esto o, que estuve en poco de caer de mi estado, no tanto de hambre como por conocer de todo en todo la fortuna serme adversa. All se me presentaron de nuevo mis fatigas y torn a llorar de mis trabajos. All se me vino a la memoria la consideracin que haca cuando me pensaba ir del clrigo, diciendo que, aunque aquel era desventurado y msero, por ventura topara con otro peor. Finalmente, all llor mi trabajosa vida pasada y mi muerte venidera. Y con todo ello, disimulando lo mejor que pude: - Seor, mozo soy, que no me fatigo mucho por comer, bendito Dios. De eso me podr yo alabar entre todos mis iguales, por de mejor garganta, y ans fui yo loado de ella hasta hoy da de los amos que yo he tenido. - Virtud es esa - dijo l- , y por eso te querr yo ms. Porque el hartar es de los puercos y el comer regaladamente es de los hombres de bien. Pseme a un cabo del portal y saqu unos pedazos de pan del seno, que me haban quedado de los de por Dios. l que vio esto, djome: - Ven ac, mozo. Qu comes? Yo llegueme a l y mostrele el pan. Tomome l un pedazo de tres que eran, el mejor y el ms grande. Y djome: - Por mi vida, que parece este buen pan. - Y cmo, agora - dije yo, seor, es bueno? - S, a fe - dijo l- . Adnde lo hubiste? Si es amasado de manos limpias? - No s yo eso. le dije, mas a m no me pone asco el sabor de ello. - As plega a Dios - dijo el pobre de mi amo. Y llevndolo a la boca, comenz a dar en l tan fieros bocados como yo en lo otro.
Lazarillo de Tormes Ctedra

1. Resume el contenido de este fragmento del Lazarillo


de Tormes.

6. Qu estilos propios de la narrativa anterior estn


presentes en el Quijote? Argumenta por qu razn se la considera la primera novela moderna.

2. Caracteriza los elementos del nivel de la historia presentes en el fragmento. Personajes. Tiempo. Acciones. Espacio.

7. Explica la estructura narrativa de esta clebre obra


cervantina, caracteriza a sus protagonistas y enuncia sus temas principales.

3. Explica qu descubre Lzaro en este episodio. De qu


mdulo o mdulos de la estructura de la obra es una constante ese descubrimiento?

8. Menciona las caractersticas de las Novelas ejemplares de


Miguel de Cervantes atendiendo a los aspectos que se indican a continuacin. Caractersticas temticas Caractersticas estructurales

4. Identifica el movimiento narrativo presente en el fragmento, extrae ejemplos de discurso valorativo y universal, y seala el recurso de la irona.

9. Qu otras obras narrativas escribi Cervantes? Indica a


qu tipo de novela renacentista pertenecen.

5. Caracteriza al protagonista del Lazarillo de Tormes y


compralo con los protagonistas de las novelas de caballeras, sentimental, morisca y bizantina.

10. Redacta una exposicin sobre el tratamiento del amor


en la narrativa renacentista.

ACTIVIDADE S

13
La apariencia barroca
La ideologa del barroco, condicionada por el contexto histrico, social y religioso, presenta una serie de rasgos que la diferencian de la de otros perodos y que se manifiestan en todas las producciones culturales. En el siguiente texto se trata uno de sus componentes esenciales.

La apariencia
No hace falta recordar que la imaginacin fue el gran componente del desarrollo cultural de la pennsula ibrica en el siglo XVII. Una sociedad de cuyo acervo cultural forman parte los milagros tiene que ser, a la fuerza, una sociedad imaginativa; incluso la cantidad de plagios, copias, versiones, variaciones, amontonamiento de citas y rplicas no hacen ms que atestiguar una actividad incesante, una reflexin minimalista, detallista, barroca, que descubre y respira en el cmulo de las infinitas posibilidades de lo que puede ser y lo que es. La gran pasin del siglo es la apariencia: la vida es sueo, desconfiemos de los sentidos, he aqu la reflexin inicial del pensamiento barroco. Esta desconfianza de los sentidos no tiene tanto que ver con la desconfianza en la percepcin, sino, al contrario, con una fe en el poder de seduccin y de comunicacin de la imagen, que imposibilita discernir entre la apariencia y la realidad. Mis ojos no me engaan respecto de lo que veo, sino respecto de la naturaleza de lo que veo. Por lo tanto el engao lo es del valor de las cosas: el oro resulta finalmente ser oropel. Precisamente por ser esta una poca transida de smbolos, el engao y el equvoco fluctan siempre en torno a un error, provocado o fortuito, en la interpretacin. El drama Del Rey abajo ninguno se construye en torno a la confusin creada por una banda que debera llevar al rey y lleva a otra persona; los consejos del hidalgo de El Buscn giran en torno a cmo engaar en la apariencia, es decir, a cmo sostener la diferencia entre lo que el espectador ve y lo que la capa sabe []. Encontramos este mismo inters en la poltica, mientras que en el mbito terico y cientfico esta inquietud por los reflejos de una realidad evasiva, se manifestar de dos formas distintas. La primera, acumulando maravillas, todo aquello que al ojo pueda, por distintos motivos, sorprender, agradar o enfadar; dicho de otro modo, haciendo recuento y acopio de todos los casos, de todos los objetos, de todas las formas que presentan una combinacin original, una excrecencia, una composicin excelsa. La segunda, intentando establecer un orden en esa multitud de apariencias, intentando crear modos de medir, de fijar y comprender las relaciones pticas con la realidad: desde la naturaleza de la luz y el funcionamiento del ojo, a la medicin de las distancias y la estimacin de las alturas. Refraccin, reflexin, sombra, proyeccin, movimientos aparentes, puntos fijos, perspectiva, gnomnica son solo una parte de los intereses de este siglo.
MATERIAL FOTOCOPIABLE / Oxford University Press Espaa, S. A.

Nuria VALVERDE El prodigio en su medida, en Madrid, ciencia y Corte, Consejera de Educacin y Cultura

1 Cul es la gran pasin del siglo XVII? En qu creencia se basa? 2 Relaciona la afirmacin anterior con el poema de Argensola de la pgina 260 del Libro del alumno. 3 Cmo se manifiesta este inters por la apariencia en el mbito tcnico y cientfico? Cita algunos ejemplos.

4 La confianza en el poder de persuasin y fascinacin de las imgenes se manifiesta en la proliferacin de otros tipos de actividades. De cules se trata? 5 Tomando en cuenta las respuestas anteriores, realiza un resumen del texto. 6 Cita algunos poemas ledos en esta unidad en los que se trate el tema del engao.

ACTIVIDADE S

13
Sonetos de Luis de Gngora
Gngora escribi sonetos de diversos temas: amorosos, de influencia petrarquista; morales, satricos y burlescos. Tambin escribi los llamados heroicos, destinados a la alabanza de amigos, grandes o reyes.

A Crdoba
Oh excelso muro, oh torres coronadas de honor, de majestad, de gallarda! Oh gran ro1, gran rey de Andaluca, de arenas nobles, ya que no doradas2! Oh frtil llano, oh sierras levantadas, que privilegia el cielo y dora el da! Oh siempre glorosa patria ma, tanto por plumas cuanto por espadas3! Si entre aquellas rinas y despojos que enriquece Genil y Dauro4 baa tu memoria no fue alimento mo, nunca merezcan mis ausentes ojos ver tu muro, tus torres y tu ro, tu llano y sierra, oh patria, oh flor de Espaa! 5

Grandes ms que elefantes y que abadas


Grandes6, ms que elefantes y que abadas7, ttulos8 liberales9 como rocas, gentiles hombres, solo de sus bocas, ilustri cavaglier10, llaves doradas11; hbitos12, capas digo remendadas, damas de haz y envs, viudas sin tocas13, carrozas14 de ocho bestias, y aun son pocas con las que tiran y que son tiradas; catarriberas15, nimas en pena, con Brtulos y Abades16 la milicia, y los derechos con espada y daga; casas17 y pechos18, todo a la malicia; lodos con perejil19 y yerbabuena: esto es la Corte. Buena pro les haga! 5

10

10

Suspiros tristes, lgrimas cansadas


Suspiros tristes, lgrimas cansadas, que lanza el corazn, los ojos llueven, los troncos baan y las ramas mueven de estas plantas5 a Alcides consagradas; mas del viento las fuerzas conjuradas los suspiros desatan y remueven, y los troncos las lgrimas se beben, mal ellos y peor ellas derramadas. Hasta en mi tierno rostro aquel tributo que dan mis ojos, invisible mano de sombra o de aire me le deja enjuto, porque aquel ngel fieramente humano no crea mi dolor, y as es mi fruto llorar sin premio y suspirar en vano. 5

por plumas espadas: se refiere a los cordobeses ilustres: Sneca, Juan de Mena y el Gran Capitn. 4 Genil y Dauro: se refiere a los dos ros de Granada.
5 6 7 8 9

plantas: lamos. grandes: ttulo nobiliario. abada: rinoceronte. ttulos: nobles. liberales: prdigos, generosos. ilustri cavaglier: en italiano, ilustres caballeros. llaves doradas: las de los del servicio de la cmara real.

10 11 12

10

hbito: se refiere a los caballeros de las rdenes militares. damas tocas: infieles y viudas alegres. 14 carrozas: las mejores solan tener cuatro caballos. 15 catarriberas: abogados, alcaldes y corregidores. 16 Brtulos y Abades: jurisconsultos del derecho civil y cannico.
13

gran ro: traduccin literal de Guadalquivir. 2 doradas: las distingue del Dauro, que llevaba oro.
1

18 19

pechos: voluntades. perejil: excrementos, inmundicias que se arrojaban a la calle.

1 Qu elementos de su ciudad natal, alaba Gngora en los cuartetos de A Crdoba? 2 Explica cmo expresa la aoranza y el amor a su tierra en los tercetos. 3 Qu recurso aparece en el ltimo terceto? 4 Segn los cuartetos del segundo texto, cmo se siente la voz potica? 5 Qu relacin se establece entre los suspiros y las lgrimas y los elementos de la naturaleza? Indica las correlaciones que hay en cada verso.

6 Quin es el ngel fieramente humano? Explica cul es la conclusin expresada en los tercetos. 7 Menciona el tema del soneto Suspiros tristes, lgrimas cansadas. 8 Qu se describe en el tercer texto? 9 Haz una relacin de los tipos humanos que componen la enumeracin de los cuartetos. 10 Qu tpico desarrolla el primer terceto? 11 Cul es la imagen final? En qu tipo de sonetos incluiras este poema?

MATERIAL FOTOCOPIABLE / Oxford University Press Espaa, S. A.

17

casas malicia: las construidas para librarse de la obligacin de alojar a los servidores del rey.

ACTIVIDADE S

13
Sonetos de Francisco de Quevedo
Se ha hecho una clasificacin temtica de la poesa de Quevedo, que incluye los poemas amorosos, los metafsicos y morales, los religiosos y los satricos y burlescos. Aqu se reproducen tres sonetos que tratan los temas ms importantes.

Vivir es caminar breve jornada


Vivir es caminar breve jornada y muerte viva es, Lico, nuestra vida, ayer al frgil cuerpo amanecida, cada instante en el cuerpo sepultada. Nada, que siendo, es poco y ser nada en poco tiempo, que ambiciosa olvida, pues de la vanidad mal persuadida anhela duracin tierra animada. Llevada de engaoso pensamiento y de esperanza burladora y ciega, tropezar en el mismo monumento, como el que divertido el mar navega y sin moverse vuela con el viento, y antes que piense en acercarse llega. 5

A Apolo siguiendo a Dafne


Bermejazo3 platero de las cumbres, a cuya luz se espulga4 la canalla, la ninfa5 Dafne, que se afufa6 y calla, si la quieres gozar, paga y no alumbres. Si quieres ahorrar de pesadumbres, ojo del cielo, trata de compralla: en confites gast Marte7 la malla8, y la espada en pasteles y en azumbres9. Volvise en bolsa10 Jpiter severo; levantse las faldas la doncella por recogerle en lluvia de dinero. Astucia fue de alguna duea11 estrella12, que de estrella sin duea no lo infiero: Febo13, pues eres sol, srvete de ella. 5

10

10

hidrpica: sedienta. reino del espanto: el Infierno. bermejazo: rojizo, pelirrojo. espulgar: limpiar de pulgas.

En los claustros de lalma la herida


En los claustros de lalma la herida yace callada, mas consume hambrienta la vida, que en mis venas alimenta llama por las medulas extendida. Bebe el ardor hidrpica1 mi vida 5 que ya, ceniza amante y macilenta, cadver del incendio hermoso ostenta su luz en humo y noche fallecida. La gente esquivo y me es horror el da; dilato en largas noches negro llanto, 10 que a sordo mar mi ardiente pena enva. A los suspiros di la voz de el canto; la confusin inunda lalma ma; mi corazn es reino del espanto2.
MATERIAL FOTOCOPIABLE / Oxford University Press Espaa, S. A.

2 3 4 5

ninfa: diosa, y en germana (jerga de ladrones y malhechores), prostituta. Marte: dios romano de la guerra. Quevedo da una versin burlesca de los amores de Marte y Venus. malla: parte de la armadura parecida a una red. azumbre: medida de capacidad. bolsa: odre, y tambin, bolsa para el dinero. Jpiter se convirti en lluvia de oro para seducir a Dnae. estrella: embustera y astro. Febo: nombre romano del dios Apolo.

afufar: huir, escapar. Voz de germana.

8 9

10

11

duea: alcahueta; mujer que no es doncella.

12 13

1 Cmo define la vida el yo potico en el primer cuarteto de Vivir es caminar breve jornada? Qu tpico literario aparece? 2 Segn el segundo cuarteto, qu desea, sin embargo, el ser humano? 3 Qu idea desarrollan los tercetos? 4 Menciona los temas del texto y reconoce los principales recursos expresivos. 5 A qu herida se refiere la voz potica en el segundo texto? Qu efectos tiene y en qu convierte su vida? 6 Cmo se comporta el yo potico? Cules son sus sentimientos?

7 Qu motivos se desarrollan en el segundo soneto? Reconoce las metforas referidas al amor.

8 Cul es el consejo que le da la voz potica


a Apolo en el tercer texto? 9 Qu ejemplos de historias mitolgicas utiliza como argumentos? Cul es la conclusin? 10 Qu visin del amor aparece en el tercer poema? 11 Analiza los principales recursos expresivos utilizados en A Apolo siguiendo a Dafne. 12 Adscribe los tres sonetos en la clasificacin temtica de la poesa de Francisco Quevedo.

ACTIVIDADE S

Lope y otros poetas barrocos

14

Adems de la obra potica de Lope de Vega, destacan en este perodo la de otros autores como Francisco de Rioja (Sevilla, h. 1583-1639) y Gabriel Bocngel (Madrid, 1603-1658), en cuyas composiciones aparecen tratados los temas ms importantes de la lrica del barroco.

Definicin de amor
Desmayarse, atreverse, estar furioso, spero, tierno, liberal, esquivo, alentado, mortal, difunto, vivo, leal, traidor, cobarde y animoso; no hallar fuera del bien centro y reposo, mostrarse alegre, triste, humilde, altivo, enojado, valiente, fugitivo, satisfecho, ofendido, receloso; huir el rostro al claro desengao, beber veneno por licor save, olvidar el provecho, amar el dao; creer que un cielo en un infierno cabe, dar la vida y el alma a un desengao: esto es amor: quien lo prob lo sabe.
Lope DE VEGA Poesa completa, Castalia

Huye del sol el sol, y se deshace


Huye del sol el sol, y se deshace la vida a manos de la propia vida; del tiempo que, a sus partos homicida, en mies de siglos las edades pace, 5 nace la vida, y con la vida nace del cadver la fbrica temida. Qu teme, pues, el hombre en la partida, si vivo estriba en lo que muerto yace? Lo que pas ya falta; lo futuro an no se vive; lo que est presente no est, porque es su esencia el movimiento. Lo que se ignora es solo lo seguro; este mundo, repblica de viento que tiene por monarca un accidente.
Gabriel BOCNGEL Poesa lrica del Siglo de Oro, Ctedra

10

10

Pura, encendida rosa


Pura, encendida rosa, mula de la llama que sale con el da, cmo naces tan llena de alegra si sabes que la edad que te da el cielo es apenas un breve y veloz vuelo? Y ni valdrn las puntas de tu rama, ni prpura hermosa a detener un punto la ejecucin del hado presurosa. El mismo cerco alado que estoy viendo riente, ya temo, amortiguado, presto despojo de la llama ardiente. Para las hojas de tu crespo seno te dio Amor de sus alas blandas plumas, y oro de su cabello dio a tu frente.
1

10

15

Francisco DE RIOJA Poesa lrica del Siglo de Oro Ctedra

de la deidad: Venus, que naci de la espuma del mar, se pinch con una espina y, al teir las rosas blancas con su sangre se volvieron rojas.

1 Qu concepcin del amor desarrolla el soneto de Lope? 2 Qu recursos expresivos utiliza para ello? 3 Segn el segundo poema, a qu est unida la vida desde su nacimiento? 4 Cul es la concepcin del tiempo y de la vida que se expone en los tercetos?

5 Qu descripcin de la rosa aparece en el poema Pura, encendida rosa, de Francisco de Rioja? 6 De qu tema importante del barroco se convirti en un smbolo la rosa? 7 Relaciona los temas de estos tres poemas con los de la lrica barroca.

MATERIAL FOTOCOPIABLE / Oxford University Press Espaa, S. A.

Oh fiel imagen suya peregrina! Bate en su color sangre divina de la deidad 1 que dieron las espumas. Y esto, purprea flor, esto no pudo hacer menos violento el rayo agudo? Rbate en una hora, rbate licencioso su ardimiento el color y el aliento: tiendes an no las alas abrasadas y ya vuelan al suelo desmayadas: tan cerca, tan unida est al morir tu vida, que dudo si en sus lgrimas la aurora mustia, tu nacimiento o muerte llora.

20

25

30

ACTIVIDADE S

La fuerza del amor, de Mara de Zayas


Mara de Zayas y Sotomayor es una de las principales representantes de la evolucin de la novela corta, dentro de la variedad de la novela cortesana, a la que aport la originalidad de un punto de vista femenino. En 1637 public sus Novelas amorosas y ejemplares, y en 1647, la Parte segunda del sarao y entretenimiento honesto, conocida tambin con el nombre de Desengaos. Las Novelas constan de un marco narrativo (las reuniones en casa de Lisis) en el que, a su vez, se desarrolla una intriga amorosa, y de una serie de relatos narrados por los personajes del marco. Los temas principales de estas historias son el amor apasionado y firme, la deshonra y la venganza que esta desencadena, los celos, el sometimiento de la mujer y la violencia que se ejerce sobre ella, as como la bsqueda de independencia y libertad por parte de las mujeres. La autora rechaza la superioridad del hombre y denuncia la marginacin y subordinacin de la mujer, a quien se le vedaba la educacin y la cultura. Cuestiona, adems, el sistema patriarcal, que impona el matrimonio como nico destino de la mujer en general y como solucin en caso de violacin. La fuerza del amor es una de las narraciones de las Novelas amorosas y ejemplares. En este relato, Laura, la protagonista, se casa con Diego, un galn muy enamorado al principio, pero que, poco despus de la boda, reanuda sus relaciones con una novia anterior y empieza a despreciar a su esposa, llegando hasta el maltrato fsico. En el siguiente fragmento la protagonista habla consigo misma.

14

La fuerza del amor


Desdichada de ti, Laura, y cmo fueras ms venturosa si como le cost tu nacimiento la vida a tu madre, fuera tambin la tuya sacrificio de la muerte! Oh amor, enemigo mortal de las gentes! Y qu de males han venido por ti al mundo, y ms a las mujeres que como en todo somos las ms perdidosas y las ms fciles de engaar, parece que solo contra ellas tienes el poder, o por mejor decir, el enojo. No s para qu el cielo me cre hermosa, noble y rica, si todo haba de tener tan poco valor contra la desdicha, sin que tantos dotes de naturaleza y fortuna me quitasen la mala estrella en que nac. O, ya que lo soy, para qu me guarda la vida?, pues tenerla un desdichado ms es agravio que ventura. A quin contar mis penas que me las remedie? Quin oir mis quejas que se enternezca? Y quin ver mis lgrimas que me las enjugue? Nadie, por cierto, pues mi padre y hermanos, por no orlas me han desamparado, y hasta el cielo, consuelo de los afligidos, se hace sordo por no drmele. Ay don Diego, y quin lo pensara! Mas s debiera pensar, si mirara que eres hombre, cuyos engaos quitan el poder a los mismos demonios y hacen ellos lo que los ministros de maldades dejan de hacer. Dnde se hallar un hombre verdadero? En cul dura la voluntad un da, y ms si se ven queridos?, que parece que al paso que conocen el amor, crece su libertad y aborrecimiento. Malhaya la mujer que en ellos cree, pues al cabo hallar el pago de su amor, como yo le hallo! Quin es la necia que desea casarse, viendo tantos y tan lastimosos ejemplos?, pues la que ms piensa que acierta, ms yerra. Cmo es mi nimo tan poco, mi valor tan afeminado y mi cobarda tanta que no quito la vida, no solo a la enemiga de mi sosiego, sino al ingrato que me trata con tanto rigor? Mas, ay, que tengo amor! Y en lo uno temo perderle, y en lo otro enojarle. Por qu, vanos legisladores del mundo, atis nuestras manos para las venganzas, imposibilitando nuestras fuerzas con vuestras falsas opiniones, pues nos negis letras y armas? El alma no es la misma que la de los hombres? Pues si ella es la que da valor al cuerpo, quin obliga a los nuestros a tanta cobarda? Yo os aseguro que si entendierais que tambin haba en nosotros valor y fortaleza, no os burlarais como os burlis. Y as, por tenernos sujetas desde que nacemos, vais enflaqueciendo nuestras fuerzas con los temores de la honra, y el entendimiento con el recato de la vergenza, dndonos por espadas, ruecas, y por libros, almohadillas. Mas triste de m! De qu me sirven estos pensamientos, pues ya no sirven para remediar cosas tan sin remedio?
Mara DE ZAYAS Y SOTOMAYOR Novelas amorosas y ejemplares Ctedra

MATERIAL FOTOCOPIABLE / Oxford University Press Espaa, S. A.

1 A quin increpa Laura al principio del texto? 2 Explica qu reproche le hace la joven a Diego, su marido, en el segundo prrafo y, por extensin, a todos los hombres. 3 A quines se dirige Laura a continuacin? Explica de qu los acusa.

4 Indica de qu recurso expresivo se vale la autora para formular los reproches de Laura. 5 Distingue en el fragmento las partes que corresponden al discurso universal y al discurso valorativo. 6 Explica la denuncia de la situacin de la mujer que se expresa al final del texto.

ACTIVIDADE S

Guzmn de Alfarache, de Mateo Alemn


El clebre fragmento que reproducimos a continuacin corresponde al principio del libro, cuando Guzmanillo, solo y todava muy inocente, deja su casa y sale a recorrer el mundo. Ha andado mucho, est cansado y hambriento, llega a una venta y pide de comer; el carcter repulsivo del episodio ilustra claramente la existencia del engao, del que es vctima por primera vez.

14

De lo que le sucedi a Guzmanillo en una venta


Djele [a la ventera] que iba a la corte, que me diese de comer. Hzome sentar en un banquillo cojo y encima de un poyo me puso un barredero de horno, con un salero hecho de un suelo de cntaro, un tiesto de gallinas lleno de agua y una media hogaza ms negra que los manteles. Luego me sac en un plato una tortilla de huevos, que pudiera llamarse mejor emplasto de huevos. Ellos, el pan, jarro, agua, salero, sal, manteles y la huspeda, todo era de lo mismo. Halleme bozal, el estmago apurado, las tripas de posta, que se daban unas con otras de vacas. Com, como el puerco la bellota, todo a hecho; aunque verdaderamente senta crujir entre los dientes los tiernecitos huesos de los sin ventura pollos, que era como hacerme cosquillas en las encas. Bien es verdad que se me hizo novedad y aun en el gusto, que no era como el de los otros huevos que sola comer en casa de mi madre; mas dej pasar aquel pensamiento con la hambre y el cansancio, parecindome que la distancia de la tierra lo causaba y que no eran todos de un sabor ni calidad. Yo estaba de manera que aquello tuve por buena suerte. Tan propio es al hambriento no reparar en salsas, como al necesitado salir a cualquier partido. Era poco, paselo presto con las buenas ganas. En el pan me detuve algo ms. Comilo a pausas, porque siendo muy malo, fue forzoso llevarlo de espacio, dando lugar unos bocados a otros, que bajasen al estmago por su orden. Comencelo por las cortezas y acabelo en el migajn, que estaba hecho engrudo; mas tal cual, no le perdon letra ni les hice a las hormigas migaja de cortesa ms que si fuera poco y bueno. As acontece si se juntan buenos comedores en un plato de fruta, que picando primero en la ms madura, se comen despus la verde, sin dejar memoria de lo que all estuvo. [] As prosegu mi camino, y no con poco cuidado de saber qu pudiera ser aquel taerme castaetas los huevos en la boca. Fui dando y tomando en esta imaginacin, que, cuanto ms la segua, ms gnero de desventuras me representaba y el estmago se me alteraba; porque nunca sospech cosa menos que asquerosa, vindolos tan mal guisados, el aceite negro, que pareca de suelos de candiles, la sartn negra y la ventera legaosa. Entre unas y otras imaginaciones, encontr con la verdad y, teniendo andada otra legua, con solo aquel pensamiento, fue imposible resistirme. Porque, como a mujer preada, me iban y venan eruptaciones del estmago a la boca, hasta que de todo punto no me qued cosa en el cuerpo. Y aun el da de hoy me parece que siento los pobreticos pollos pindome ac dentro. As estaba sentado en la falda del vallado de unas vias, considerando mis infortunios, harto arrepentido de mi mal considerada partida, que siempre se despean los mozos tras el gusto presente, sin respetar ni mirar el dao venidero.
Mateo ALEMN Guzmn de Alfarache, Planeta

1 Qu condiciones presenta la mesa que le sirven en la venta al protagonista? 2 Qu plato le sirve la ventera? Explica qu come en primer lugar y qu piensa el protagonista del estado del ingrediente principal.

3 El protagonista sigue su camino: Qu efectos le causan los alimentos? Qu verdad descubre? 4 Explica la reflexin final del narrador. 5 Seala en este fragmento enunciados de discurso valorativo y de discurso universal.

MATERIAL FOTOCOPIABLE / Oxford University Press Espaa, S. A.

ACTIVIDADE S

Una descripcin mordaz


El siguiente texto corresponde a la famosa descripcin del licenciado Cabra, en la que Quevedo muestra su ingenio y su capacidad para la agudeza. Pablos, el protagonista, marcha a Segovia como criado de don Diego Coronel, a quien su padre ha puesto bajo la tutela del licenciado.

14

El licenciado Cabra
Entramos, primero domingo despus de Cuaresma, en poder de la hambre viva, porque tal laceria1 no admite encarecimiento. l era un clrigo cerbatana, largo2 solo en el talle; una cabeza pequea; los ojos, avecindados en el cogote, que pareca que miraba por cuvanos3, tan hundidos y escuros, que era buen sitio el suyo para tiendas de mercaderes; la nariz, de cuerpo de santo, comido el pico, entre Roma y Francia4, porque se le haba comido de unas bas5 de resfriado, que aun no fueron de vicio porque cuestan dinero; las barbas, descoloridas de miedo de la boca vecina, que, de pura hambre, pareca que amenazaba a comrselas; los dientes, le faltaban no s cuntos, y pienso que por holgazanes y vagamundos se los haban desterrado; el gaznate, largo como de avestruz, con una nuez tan salida, que pareca se iba a buscar de comer forzada de la necesidad; los brazos, secos; las manos, como un manojo de sarmientos cada una; mirado de medio abajo, pareca tenedor o comps, con dos piernas largas y flacas; su andar, muy espacioso: si se descompona algo, le sonaban los gesos como tablillas de San Lzaro6; la habla, tica7; la barba, grande, que nunca se la cortaba por no gastar, y l deca que era tanto el asco que le daba ver la mano del barbero por su cara, que antes se dejara matar que tal permitiese: cortbale los cabellos un muchacho de nosotros. Traa un bonete los das de sol, ratonado con mil gateras, y guarniciones de grasa; era de cosa que fue pao, con los fondos en caspa. La sotana, segn decan algunos, era milagrosa, porque no se saba de qu color era.Unos,vindola tan sin pelo,la tenan por de cuero de rana; otros decan que era ilusin: desde cerca pareca negra, y desde lejos entreazul. Llevbala sin ceidor; no traa cuello ni puos. Pareca, con esto y los cabellos largos y la sotana y el bonetn, teatino lanudo8. Cada zapato poda ser tumba de un filisteo9. Pues su aposento? Aun araas no haba en l. Conjuraba los ratones de miedo que no le royesen algunos mendrugos que guardaba. La cama tena en el suelo, y dorma siempre de un lado por no gastar las sbanas. Al fin, l era archipobre y protomiseria.
Francisco DE QUEVEDO La vida del Buscn, Crtica

1 2 3

laceria: miseria.

largo: alto y, tambin, generoso (diloga).

cuvanos: cestos de mimbre usados en la vendimia.

tablillas de san Lzaro: las utilizaban las personas enfermas de lepra para avisar de su presencia y pedir limosna. tica: un tipo de fiebre, que afectaba al habla.

7 8

MATERIAL FOTOCOPIABLE / Oxford University Press Espaa, S. A.

entre Roma y Francia: alude a que la nariz era roma (pequea y poco puntiaguda) y deforme, como atacada por la sfilis (llamada el mal francs).

teatino lanudo: religioso de una orden que sola acompaar a los condenados a muerte. filisteo: hombre de gran estatura y corpulencia.

bas: bubas, tumores blandos.

1 Divide el texto en partes segn los aspectos que abarca la descripcin: partes del cuerpo, vestimenta, habitacin y hbitos. 2 Que rasgo principal se destaca de la conducta del licenciado Cabra? 3 Qu personaje de una novela picaresca anterior tiene caractersticas similares a las del licenciado?

4 Analiza los recursos expresivos y determina cules predominan. 5 Realiza una parfrasis del texto. 6 Relaciona el modo en que se describe al personaje con los rasgos temticos y formales de la obra. 7 Reconoce algunas de las caractersticas propias de un texto descriptivo.

ACTIVIDADE S

Los Sueos, de Quevedo


En El alguacil endemoniado el narrador habla con un demonio que se ha apoderado del alma de un alguacil, personaje que pertenece a un sector social habitualmente criticado por Quevedo; el diablo advierte que estos golpes que le doy y lo que le aporreo, no es sino que yo y l reimos ac sobre quin ha de estar en mejor lugar y andamos a ms diablo es l. El narrador le pregunta sobre quines estn en el Infierno y sobre sus penas.

14

Del sitio que a cada cual le corresponde en el Infierno


Y en el Infierno estn todos aposentados as. Que un artillero que baj all el otro da, queriendo que le pusiesen entre la gente de guerra, como al preguntarle del oficio que haba tenido dijese que hacer tiros1 en el mundo, fue remitido al cuartel de los escribanos, pues son los que hacen tiros en el mundo. Un sastre, porque dijo que haba vivido de cortar de vestir2, fue aposentado con los maldicientes. Un ciego, que quiso encajarse con los poetas, fue llevado a los enamorados, por serlo todos.Los que venan por el camino de los locos, ponemos con los astrlogos, y a los por mentecatos, con los alquimistas. Uno vino por unas muertes, y est con los mdicos. Los mercaderes que se condenan por vender, estn con Judas. Los malos ministros, por lo que han tomado, alojan con el mal ladrn. Los necios estn con los verdugos.Y un aguador, que dijo haba vendido agua fra, fue llevado con los taberneros. Lleg un mohatrero3 tres das ha, y dijo que l se condenaba por haber vendido gato por liebre, y pusmoslo de pies con los venteros, que dan lo mismo. Al fin, el Infierno est repartido en estas partes. Ote decir antes de los enamorados, y por ser cosa que a m me toca, gustara saber si hay muchos. Mancha es la de los enamorados respondi que lo toma todo, porque todos lo son de s mismos: algunos, de sus dineros; otros de sus palabras; otros, de sus obras, y algunos, de las mujeres. Y destos postreros hay menos que de todos en el Infierno, porque las mujeres son tales, que, con ruindades, con malos tratos y peores correspondencias les dan ocasiones de arrepentimiento cada da a los hombres. [] Algunos hay que en celos y esperanzas amortajados y en deseos, se van por la posta4 al Infierno, sin saber cmo ni cundo ni de qu manera.
MATERIAL FOTOCOPIABLE / Oxford University Press Espaa, S. A.

Francisco DE QUEVEDO Sueos, Espasa Calpe


1 2 3 4

hacer tiro: engaar.

cortar de vestir: murmurar, hablar mal de alguien. mohatrero: persona que engaa, que comete fraudes. por la posta: de prisa, velozmente.

1 Menciona todos los tipos humanos y sociales que se nombran en el primer prrafo. 2 Cmo se ubica en el Infierno a los que van llegando? Explica alguna de las distribuciones que se realizan. 3 Menciona cul es la finalidad de Quevedo al determinar la agrupacin de los condenados. Reconoce los recursos expresivos que le sirven para tal objetivo.

4 Seala los tipos de enamorados. Qu significa la frase todos lo son de s mismos? 5 Por qu hay pocos enamorados de las mujeres en el Infierno? 6 Relaciona el contenido del texto con los temas de los Sueos. 7 En qu otras obras de Quevedo aparecen temas semejantes?

ACTIVIDADE S

Orculo manual y arte de prudencia, de Baltasar Gracin


Este libro, compuesto por trescientos aforismos, ofrece un arte de prudencia, unas normas de vida constituidas por afirmaciones caracterizadas por su laconismo. A continuacin reproducimos algunos de ellos.

14

105. No cansar. Suele ser pesado el hombre de un negocio1 y el de un verbo. La brevedad es lisonjera, y ms negociante: gana por lo corts lo que pierde por lo corto. Lo bueno, si breve, dos veces bueno; y aun lo malo, si poco, no tan malo. Ms obran quintas esencias que frragos; y es verdad comn que hombre largo raras veces es entendido, no tanto en lo material de la disposicin, cuanto en lo formal del discurso. Hay hombres que sirven ms de embarazo que de adorno del universo, alhajas perdidas que todos las desvan. Excuse el discreto el embarazar, y mucho menos a grandes personajes, que viven muy ocupados; y sera peor desazonar uno de ellos que todo lo restante del mundo. Lo bien dicho se dice presto.

174. No vivir aprisa. El saber repartir las cosas es saberlas gozar. A muchos les sobra la vida y se les acaba la felicidad; malogran los contentos, que no los gozan, y querran despus volver atrs cuando se hallan tan adelante; postillones2 del vivir, que, a ms del comn correr del tiempo, aaden ellos su atropellamiento genial; querran devorar en un da lo que apenas podran digerir en toda la vida; viven adelantados en las felicidades, cmense los aos por venir y, como van con tanta prisa, acaban presto con todo. Aun en el querer saber ha de haber modo, para no saber las cosas mal sabidas. Son ms los das que las dichas. En el gozar, a espacio; en el obrar, aprisa. Las hazaas, bien estn hechas; los contentos, mal acabados.

195. Saber estimar. Ninguno hay que no pueda ser maestro de otro en algo; ni hay quien no exceda al que excede. Saber disfrutar a cada uno es til saber. El sabio estima a todos, porque reconoce lo bueno en cada uno, y sabe lo que cuestan las cosas de hacerse bien. El necio desprecia a todos por ignorancia de lo bueno y por eleccin de lo peor.

MATERIAL FOTOCOPIABLE / Oxford University Press Espaa, S. A.

246. Nunca dar satisfaccin a quien no la peda. Y aunque se pida, es especie de delito si es sobrada. El excusarse antes de ocasin es culparse, y el sangrarse en salud es hacer del ojo 3 al mal y a la malicia. La excusa anticipada despierta al recelo que dorma. Ni se ha de dar el cuerdo por entendido de la sospecha ajena, que es salir a buscar el agravio; entonces le ha de procurar desmentir con la entereza de su proceder.

299. Dejar con hambre. Hase de dejar en los labios aun con el nctar. Es el deseo medida de la estimacin. Hasta la material sed es treta de buen gusto picarla, pero no acabarla: lo bueno, si poco, dos veces bueno. Es grande la baja de la segunda vez 4. Hartazgos de agrado son peligrosos, que ocasionan desprecios a la ms eterna eminencia. nica regla de agradar, coger el apetito picado con el hambre con que se qued. Si se ha de irritar, sea antes por impaciencia del deseo que por enfado de la fruicin; gstase al doble de la felicidad penada.
Baltasar GRACIN Orculo manual y arte de prudencia Planeta

1 2

negocio: asunto, ocupacin.

3 4

hacer del ojo: avisar, llamar la atencin. es grande la baja de la segunda vez: la segunda vez que algo se aprecia baja en estimacin.

postilln: mozo que iba a caballo, delante de las postas para guiar y ensear el camino.

1 El primer aforismo contiene un enunciado muy difundido, cul es? A qu situacin lo aplica el texto? Pon ejemplos de otros casos en que tambin podra utilizarse esta expresin. 2 Establece relaciones temticas entre los aforismos 105 y 299 y localiza en ambos una expresin casi idntica.

3 Cules son los argumentos que fundamentan los consejos de No vivir aprisa y Saber estimar? 4 Por qu conviene no excusarse de lo que uno no ha sido acusado? Qu consecuencias trae? 5 Redacta un texto sobre el vivir deprisa en la sociedad actual (cmo se manifiesta y en quines, cules son sus motivos y justificaciones, qu juicios te merecen, etc.).

Evaluacin 14

Alegora de una poca fugaz, por Antoni da Pereda.

Huye sin percibirse lento el da


Huye sin percibirse lento el da, y la hora secreta1 y recatada con silencio se acerca, y despreciada2 lleva tras s la edad lozana ma. La vida nueva, que en niez arda, la juventud robusta y engaada, en el postrer invierno3 sepultada, yace entre negra sombra y nieve fra. No sent resbalar mudos los aos; hoy los lloro pasados y los veo riendo de mis lgrimas y daos. Mi penitencia deba a mi deseo, pues me deben la vida mis engaos4 y espero el mal que paso y no le creo.

1. Reconoce el gnero al que pertenece el texto y justifica


los motivos de esa adscripcin.

2. Explica el contenido del poema y determina el o los


temas que desarrolla.

3. Qu motivo propio de la poesa de Quevedo aparece


5 en este texto? En qu tipos de obras lo trata?

4. Realiza el anlisis mtrico de la composicin. Qu otras


formas mtricas se cultivaron en el barroco?

5. Analiza los recursos expresivos ms destacados del texto.


10

6. Enuncia y explica otros tipos de poesa que haya cultivado


Quevedo y menciona las caractersticas de su estilo.

7. Realiza una sntesis de la obra potica de Gngora y


de las caractersticas estilsticas ms importantes.

Francisco DE QUEVEDO Poesa completa, Planeta

8. Menciona las obras principales de Lope de Vega, los


temas tratados en general y la particularidad de su estilo.

9. Relaciona los temas de la poesa y la prosa barrocas con


el contexto histrico, social y literario de su produccin.
1 2 3 4

secreta: ignorada, escondida. despreciada: la juventud (la edad lozana). postrer invierno: la vejez. me deben la vida mis engaEos: los engaos le han quitado la vida.

10. Redacta un texto en que reflexiones, en general, sobre


la brevedad de la vida y su final inevitable (por ejemplo, si has pensado en ello alguna vez o es una realidad percibida como muy lejana, cmo afrontarla, si compartes la actitud de los poetas barrocos ).

ACTIVIDADE S

1. Auto de los Reyes Magos


El Auto o Representacin de los Reyes Magos, perteneciente al ciclo del Ordo Stellae (escenificado en las fiestas de la Epifana), es el ms antiguo drama sacro en romance que se conserva. Consta de los monlogos de los Reyes, su encuentro y la decisin de peregrinar juntos, la visita a Herodes y el dilogo de este con sus consejeros.

Encuentro de los Magos con Herodes


GASPAR.(Dirigindose a HERODES, que se acaricia la negra y breve barba con gesto impaciente.) Gurdete el Creador. Dios te libre de mal. Te entretendremos poco Un instante no ms. (HERODES, apresurado, le estimula a continuar. GASPAR considera prudente repetir el saludo.) Dios te d larga vida, y te libre de mal. Vamos en romera, para a un rey adorar, que ha nacido en la tierra No logrmosle hallar. HERODES.(Sbitamente interesado.) Qu decs? Dnde vais? A quin vais a buscar? De qu tierra vens? Dnde intentis llegar? Decidme vuestros nombres, no los queris celar. GASPAR.Yo me llamo Gaspar; este que ves, Melchor; y aquel es Baltasar. (GASPAR no interpreta bien el inters de HERODES. Atropellado e ingenuo le comunica la nueva.) Rey, un rey ha nacido que es seor de la tierra, que mandar en el mundo con gran paz y sin guerra! HERODES.Es as en verdad? GASPAR.S, rey, por caridad. HERODES.Y cmo lo sabis? Probado lo tenis? GASPAR.Rey, verdad te diremos: probado lo tenemos. MELCHOR.(Sin poder contenerse, seala hacia el cielo.) Esto es gran maravilla: un astro nuevo brilla!
MATERIAL FOTOCOPIABLE / Oxford University Press Espaa, S. A.

BALTASAR.Seal de que ha nacido y que en carne ha venido! (Nueva pausa. La voz de HERODES tiembla levemente, mientras mira a lo alto.) HERODES.Cunto hace que le visteis y que lo percibisteis? GASPAR.Trece das son ya. (Volvindose a sus compaeros.) Creo que ms no har desde que est advertido y muy bien percibido. HERODES.(Con tono que sera amable si no le delatara el temblor de la voz.) Pues andad y buscad, y a ese rey adorad, y por aqu tornad (Ligera pausa. El REY no mira ya a los MAGOS.) Si lo encontris, ir, y yo le adorar. (Se apaga la luz del espacio central. GASPAR sale, tras haberse inclinado ante HERODES. Este, solo, se pone en pie. Ya no tiene por qu disimular su clera.) Quin vio nunca tal mal? Sobre un rey, otro tal! An no estoy yo muerto ni bajo tierra puesto. Otro rey sobre m? Nunca tal cosa vi! El mundo va hacia atrs: no s qu me haga ya. Juro que no lo crea hasta que no lo vea.
Teatro medieval Castalia

1 Explica por qu el fragmento del Auto de los Reyes Magos pertenece a un texto dramtico. 2 Resume el contenido del dilogo que mantienen los personajes. Cul es el conflicto que presenta? Explcalo brevemente. 3 Caracteriza los personajes que intervienen en este pasaje atendiendo especialmente a sus palabras.

4 Las acotaciones fueron incorporadas por el profesor Fernando Lzaro Carreter, en la edicin que prepar del Auto. Seala a qu aspectos de los cdigos visuales y auditivos de la representacin hacen referencia. 5 Reconoce en las intervenciones de los personajes que participan en el dilogo, ejemplos de las funciones del lenguaje expresiva, apelativa y referencial.

17.

El teatro: de los orgenes a la comedia nacional

297

ACTIVIDADE S

1 7 2. Representacin del nacimiento


AMPLIACIN

de Nuestro Seor
El escritor Gmez Manrique compuso hacia 1460 la Representacin del nacimiento de Nuestro Seor, para su hermana doa Mara Manrique, vicaria en el convento de clarisas de Calabazanos (Palencia). Para esta composicin dramtica, el poeta se basa en los evangelios cannicos y emplea formas mtricas de la poesa cancioneril del siglo XV.

La desconfianza de Jos
LO QUE DICE JOS, SOSPECHANDO DE NTRA. SEORA: Oh viejo desventurado! Negra dicha fue la ma, en casarme con Mara por quien fuese deshonrado. Yo la veo bien preada; no s de quin ni de cunto. Dicen que d Espritu Santo, mas yo d esto no s nada. LA ORACIN QUE HACE LA GLORIOSA: Mi solo Dios verdadero, cuyo ser es inmovible, 1 0 a quien es todo posible, fcil y bien facedero1! T que sabes la pureza de la mi virginidad, alumbra la ceguedad 1 5 de Jos y su simpleza. EL NGEL A JOS: Oh viejo de muchos das, en el seso de muy pocos, el principal de los locos! T no sabes que Isaas 2 0 dijo: Virgen parir, lo cual escribi por esta doncella gentil, honesta, cuyo par nunca ser? LA QUE REPRESENTA A LA GLORIOSA, CUANDO LE DIEREN EL NIO: Adrote, rey del cielo, verdadero Dios y hombre; adoro tu santo nombre, mi salvacin y consuelo. Adrote, hijo y padre, a quien sin dolor par, porque quisiste de m hacer de sierva tu madre. Bien podr decir aqu aquel salmo gloroso que dije, hijo precioso, cuando yo te conceb: que mi nima2 engrandece a ti, mi solo seor, y en ti, mi salvador, mi espritu florece. Mas este mi gran placer en dolor ser tornado, pues t eres enviado para muerte padecer por salvar los pecadores en la cual yo pasar, no mengundome la fe, innumerables dolores. 25

30

35

40

45

Gmez MANRIQUE Representacin del nacimiento de Nuestro Seor en Ronald E. SURTZ, Teatro castellano de la Edad Media Taurus (actualizacin)

facedero: hacedero; que se puede hacer.

nima: alma.

1 Explica en qu consiste el conflicto que se plantea en el inicio de este fragmento de la Representacin de nacimiento de Nuestro Seor. 2 Cmo caracteriza a Jos la Virgen Mara en su primera intervencin? Qu le reprocha el ngel?

3 Identifica el tipo de estrofa empleado en este fragmento dramtico. 4 Incorpora al texto algunas acotaciones que se refieran a distintos elementos de los cdigos que intervienen en una representacin teatral (escenografa, gestos, movimientos).

17.

El teatro: de los orgenes a la comedia nacional

298

MATERIAL FOTOCOPIABLE / Oxford University Press Espaa, S. A.

UNA GLOGA DE JUAN DEL ENCINA


En el siguiente fragmento corresponde al comienzo de la gloga en la que Mengo y un escudero pugnan por el amor de la pastorcilla Pascuala.

ACTIVIDADE S

14

gloga representada en requesta1 de unos amores


MINGO.Pascuala, Dios te mantenga. PASCUALA.Nora buena vengas, Mingo. Oy, qu es da de domingo, no ests con tu esposa Menga? MINGO.No ay quien all me detenga, quel cario que te tengo me pone un quejo2 tan luengo que me acossa que me venga. PASCUALA.Y no praga3 a Dios contigo, y aun con tu esposa Menguilla! Cmo dexas tu esposilla por venirte ac comigo? MINGO.Soncas4, soncas, no te digo que eres, zagala, tan bella que te quiero ms que a ella? Dios lo sabe, qu es testigo. PASCUALA.Miaf 5, Mingo, no te creo que de m ests namorado. Pues eres ya desposado, tu querer no lo desseo. MINGO.Ay, Pascuala, que te veo tan lozana y tan garrida, que yo te juro a mi vida que deslumbro si te oteo! Y porque eres tan hermosa te quiero; mira, vers, quireme, quireme ms, pues por ti dejo a mi esposa. Y toma, toma esta rosa que para ti la cog, aunque no curas de m ni por m se te da cosa. PASCUALA.O, qu chapados olores! Mingo, Dios te d salud, y gozes la juventud ms que todos los pastores.
1 2 3 4

10

15

20

25

30

35

MINGO.Y t dasme mil dolores. Dame, dame una manija, o siquiera essa sortija, que traya por tus amores. PASCUALA.Tirte6, tirte all, Minguillo, no te quellotres7 de vero! Hete8, viene un escudero; vea que eres pastorcillo. Sacude tu caramillo9, tu hondijo10 y tu cayado; haz que aballas11 el ganado, silva, hurria12, da gritillo. ESCUDERO.Pastora, slvete Dios. PASCUALA.Dios os d, seor, buen da. ESCUDERO.Guarde Dios tu galana. PASCUALA.Escudero, ass haga a vos. ESCUDERO.Tienes ms gala que dos de las de mayor beldad. PASCUALA.Essos que sois de ciudad perchufis13 huerte14 de nos. ESCUDERO.Desso no tengas temor. Por mi vida, pastorcica, que te haga presto rica PASCUALA.Essas trnicas15, seor, all para las de villa. ESCUDERO.Vete comigo, carilla16. Dexa, dexa esse pastor. Dxalo, que Dios te vala; no te pene su penar, que no te sabe tratar segn requiere tu gala. MINGO.Estate queda, Pascuala, no te engae este traidor, palaciego, burlador, que ha burlado otra zagala.

40

45

50

55

60

65

70

MATERIAL FOTOCOPIABLE / Oxford University Press Espaa, S. A.

Juan DEL ENCINA Teatro y poesa, Taurus


9

requesta: desafo. quejo: dolor, pena. praga: agrade. soncas: en verdad, a fe.

5 6 7 8

miaf: por mi fe, s. tirte: aprtate. quellotres: lamentes. hete: mira.

caramillo: flauta de caa. hondijo: honda. aballas: arreas. hurria: arrea.

13 14 15 16

perchufis: burlis. huerte: fuerte, mucho. trnicas: retricas. carilla: compaera.

10 11 12

1 Por qu puede adscribirse este texto de Juan del Encina al gnero dramtico? 2 Resume el contenido del fragmento. 3 Caracteriza los personajes y sus actitudes. 4 Analiza la mtrica empleada.

5 Caracteriza el habla de los tres personajes de esta gloga. 6 Reconoce en el texto un recurso caracterstico de la poesa cancioneril; quin lo emplea? 7 Compara estos pastores con los de las glogas de Garcilaso de la Vega.

17.

El teatro: de los orgenes a la comedia nacional

299

ACTIVIDADE S

AMPLIACIN

Un paso de Lope de Rueda


Lope de Rueda fue el principal creador de entremeses, escenas cmicas con autonoma dramtica, conocidas en su poca como pasos. Estas obras breves se escribieron para ser representadas en un espectculo ms amplio, como una comedia. De Lope de Rueda se conservan ms de veinte pasos, que se suelen clasificar en cuatro grupos, segn el tipo de protagonista: los de simple, los de negra, los de lacayo fanfarrn y los de otras figuras. El siguiente es un fragmento del paso Las aceitunas.

Las aceitunas
ALOJA.Qus esto, vecinos? Por qu me maltratis ans la mochacha? GUEDA.Ay, seor! Este mal hombre que me quiere dar las cosas a menos precio y quiere echar a perder mi casa. Unas aceitunas que son como nueces! TORUVIO.Yo juro a los huesos de mi linaje que no son ni aun como piones. GUEDA.S son! TORUVIO.No son! ALOJA.Ora, seora vecina, haceme tamao placer que os entris all dentro, que yo lo averiguar todo. GUEDA.Averige o pngase todo del quebranto1. ALOJA.Seor vecino, qu son de las aceitunas? Sacaldas ac fuera, que yo las comprar, aunque sean veinte hanegas2. TORUVIO.Que no, seor, que no es desa manera que vuesa merced se piensa; que no estn las aceitunas aqu en casa, sino en la heredad3. ALOJA.Pues traeldas aqu, que yos las comprar todas al precio que justo fuere. MENCIGELA.A dos reales quiere mi madre que se venda el celemn4. ALOJA.Cara cosa es esa. TORUVIO.No le paresce a vuesa merced?
1

MENCIGELA.Y mi padre a quince dineros. ALOJA.Tenga yo una muestra dellas. TORUVIO.Vlame Dios, seor! Vuesa merced no me quiere entender. Hoy he yo plantado un renuevo5 de aceitunas y dice mi mujer que de aqu a seis o siete aos llevar cuatro o cinco hanegas de aceituna y quella la cogera y que yo la acarrease y la mochacha la vendiese. Y que, a la fuerza de drecho6, haba de pedir a dos reales por cada celemn. Yo, que no, y ella, que s. Y sobre esto ha sido la cuistin. ALOJA.Oh, qu graciosa cuistin! Nunca tal se ha visto. Las aceitunas no estn plantadas y ha llevado la mochacha tarea sobre ellas. MENCIGELA.Qu le paresce, seor? TORUVIO.No llores, rapaza; la mochacha, seor, es como un oro. Ora andad, hija, y ponedme la mesa, que yos prometo de hacer un ayuelo de las primeras aceitunas que se vendieren. ALOJA.Ora, andad, vecino; entraos all dentro y ten paz con vuestra mujer. TORUVIO.Adis, seor. ALOJA.Ora, por cierto, qu cosas vemos en esta vida que ponen espanto! Las aceitunas no estn plantadas, ya las habemos visto reidas. Razn ser que d fin a mi embajada7.
Lope DE RUEDA Pasos completos, Espasa Calpe
MATERIAL FOTOCOPIABLE / Oxford University Press Espaa, S. A.

averige del quebranto: esta frase se ha interpretado como averige o djese todo a favor de la prdida econmica.

celemn: medida de algo ms de cuatro litros y medio. 5 renuevo: vstago que echa una planta podada.
6 7

hanegas: fanegas, medida de capacidad equivalente a 55 litros y medio. heredad: posesin en el campo.

a la fuerza de drecho: por derecho. embajada: se llamaba as a la presentacin y embajador al presentador.

1 Justifica por qu el fragmento de Lope de Rueda pertenece a un texto dramtico. 2 Identifica los personajes que intervienen en este dilogo sealando la relacin que existe entre ellos. 3 Por qu acude Aloja? Explica cul es el conflicto.

4 Clasifica el paso Las aceitunas en uno de los grupos sealados en la explicacin que aparece ms arriba. 5 Atendiendo a las palabras finales de Aloja, sobre qu crees que tratar la comedia que se puede representar despus de este paso? Razona tu respuesta.

300 17.

El teatro: de los orgenes a la comedia nacional

14
Rasgos de la comedia nacional
Repasa los rasgos de la comedia nacional y luego lee atentamente el siguiente fragmento de La dama duende, de Caldern de la Barca.

El honor familiar
LUIS.No hay accin que me suceda bien, Rodrigo. Si una dama veo airosa, y conocerla solicito, me detienen un necio y una pendencia; que no s cual es peor; si rio, y mi hermano llega, es mi enemigo su amigo; si por disculpa me deja de una dama, es una dama que mil pesares me cuesta: de suerte que una tapada me huye, un necio me atormenta, un forastero me mata, y un hermano me le lleva a ser mi husped a casa, y otra dama me desprecia De mal anda mi fortuna! RODRIGO.Qu de todas esas penas que s la que sientes ms? LUIS.No sabes. RODRIGO. Que la que llegas a sentir ms, son los celos de tu hermana y Beatriz bella? LUIS.Engaste. RODRIGO. Pues, cul es? 2 5 de ti solo me fiara, lo que ms siento es que sea mi hermano tan poco atento, que llevar a casa quiera un hombre mozo, teniendo, 3 0 Rodrigo, una hermana en ella, viuda y moza; y, como sabes, tan de secreto, que apenas sabe el sol que vive en casa; porque Beatriz, por ser deuda, 3 5 solamente la visita. RODRIGO.Ya s que su esposo era administrador de puertos de mar en unas reales rentas, y qued debiendo al rey grande cantidad de hacienda; y ella a la corte se vino de secreto, donde intenta, LUIS.Si tengo de hablar de veras escondida y retirada, componer mejor sus deudas. Y esto disculpa a tu hermano, pues, si mejor consideras que su estado no le da ni permisin ni licencia de que nadie la visite, y que, aunque tu husped sea don Manuel, no ha de saber que en casa, seor, se encierra tal mujer, qu inconveniente hay en admitirle en ella? Y ms habiendo tenido tal recato y advertencia, que para su cuarto ha dado por otra calle la puerta, y la que sala a la casa, por desmentir la sospecha, de que el cuidado la haba cerrado, o porque pudiera otra vez, fabric en ella una alacena de vidrios, labrada de tal manera, que parece que jams en tal parte ha habido puerta. LUIS.Ves con lo que me aseguras? Pues con eso mismo intentas darme muerte, pues ya dices que no ha puesto por defensa de su honor ms que unos vidrios, que al primer golpe se quiebran. (Vanse y salen DOA NGELA e ISABEL.) DOA NGELA.Vulveme a dar, Isabel, esas tocas; pena esquiva!, vuelve a amortajarme viva, ya que mi suerte crel lo quiere as. ISABEL. Toma presto; porque si tu hermano viene y alguna sospecha tiene, no la confirme con esto, de hallarte desta manera, que hoy en palacio te vio. 45

50

10

55

15

60

20

65

70

75

MATERIAL FOTOCOPIABLE / Oxford University Press Espaa, S. A.

80

Pedro CALDERN DE LA BARCA La dama duende

40

300 17.

El teatro: de los orgenes a la comedia nacional

ACTIVIDADE S

Acotaciones de una comedia cortesana


En 1651, Baccio del Bianco realiz en el parque del Coliseo del Buen Retiro de Madrid un excepcional montaje de La fiera, el rayo y la piedra, de Caldern de la Barca. Las siguientes son algunas de las acotaciones incluidas en esta comedia cortesana.

14

Texto 1
(Oscurcese el teatro, que ser de peascos con el foro de marina; y mientras se dicen los primeros versos, se descubre la perspectiva del mar, y habr truenos y relmpagos.)

Texto 6
(Al entrarse por otra parte huyendo, salen PIGMALEN y LEBRN.)

Texto 10
(Vuela CUPIDO, mdase el teatro en el de monte, y en el foro, la fragua de VULCANO; y salen por una parte LISI, CLORI, LAURA e ISABELLA, con arcos y flechas y varios instrumentos en las manos; y por la otra ANAJARTE, en traje de cazadora, con venablo, y OTRAS.)

Texto 7 Texto 2
(Sale IRIFILE, vestida de pieles, suelto el cabello.) (Mdase el teatro en el bosque, y en el foro la gruta de LAS PARCAS.)

Texto 11
(Pnense las cuatro a las cuatro puntas del tablado, retranse ANAJARTE y las otras damas, y mientras cantan, sale IRIFILE, acechando.) (Suenan dentro los martillos de la fragua.)

Texto 3
(Atraviesan varios peces por la marina.)

Texto 8
(brese la gruta y vese en lo ms lejos de ella a LAS TRES PARCAS; la primera con una rueca, cuyo hilo va a dar a la tercera, que lo devana, dejando en medio a la segunda con unas tijeras en la mano.)

Texto 4 (Pasan algunas sirenas, cantando.)

Texto 12
(Descbrese la fragua, y los CCLOPES cantan al son de los martillos.) (Desaparece la fragua y pasa en una nube ANTEROS, atravesando el teatro con un venablo en la mano.)
Pedro CALDERN DE LA BARCA La fiera, el rayo y la piedra Ibero-Americana de Publicaciones

Texto 5
(Atraviesan algunos bajelillos por la marina.) (Empieza a aclarar.)

Texto 9
(Cantando, en tono muy triste.) (Cirrase la gruta.)

1 Todas estas acotaciones estn incluidas en La fiera, el rayo y la piedra, comedia cortesana de Caldern de la Barca. Seala aquellas posturas referidas a cuestiones relacionadas con los personajes (gestos, movimientos, vestuario). 2 Cules de estas acotaciones hacen mencin de elementos relacionados con la utilera de los personajes? Y con la utilera de escena?

3 Seala qu indicaciones se ofrecen acerca de la ambientacin de la obra. Diferencia entre: Elementos visuales. Elementos auditivos. 4 Despus de analizar estas acotaciones, relaciona su informacin con lo que sabes acerca del teatro barroco, especialmente el teatro cortesano.

MATERIAL FOTOCOPIABLE / Oxford University Press Espaa, S. A.

ACTIVIDADE S

14
Los autos sacramentales
Recuerda las caractersticas de los autos sacramentales y luego lee el siguiente fragmento, que corresponde al final de El gran teatro del mundo, de Pedro Caldern de laBarca.

El autor y sus personajes


RICO.Si el poder y la hermosura por aquella vanagloria que tuvieron, con haber llorado, tanto te asombran, y el labrador que a gemidos enterneciera una roca est temblando de ver la presencia poderosa de la vista del Autor, cmo oso mirarla ahora? Mas es preciso llegar, pues no hay adonde me esconda de su riguroso juicio. Autor! AUTOR. Cmo as me nombras? Que aunque soy tu Autor, es bien que de decirlo te corras, pues que ya en mi compaa no has de estar. De ella te arroja mi poder. Desciende adonde te atormente tu ambiciosa condicin eternamente entre penas y congojas. RICO.Ay de m! Que envuelto en fuego caigo arrastrando mi sombra donde ya que no me vea yo a m mismo, duras rocas sepultarn mis entraas en tenebrosas alcobas. DISCRECIN.Infinita gloria tengo. HERMOSURA.Tenerla espero dichosa. LABRADOR.Hermosura, por deseos no me llevars la joya. RICO.No la espero eternamente. NIO.No tengo, para m, gloria. AUTOR.Las cuatro postrimeras son las que presentes notan vuestros ojos, y porque destas cuatro se conozca que se ha de acabar la una, suba la Hermosura ahora con el Labrador, alegres, a esta mesa misteriosa, pues que ya por sus fatigas merecen grados de gloria.

10

15

(Suben los dos.) HERMOSURA.Qu ventura! LABRADOR. RICO.Qu desdicha! Qu consuelo! Qu victoria! Qu alivio! Qu ponzoa! 50 45

20

REY. DISCRECIN. POBRE.Qu dulzura!

RICO.Qu sentimiento!

25

RICO.

NIO.Gloria y pena hay, pero yo no tengo pena ni gloria. AUTOR.Pues el ngel en el cielo, en el mundo las personas y en el infierno el demonio todos a este Pan se postran; en el infierno, en el cielo y mundo a un tiempo se oigan dulces voces que le alaben acordadas y sonoras. (Tocan chirimas, cantando el Tantum ergo muchas veces.) MUNDO.Y pues representaciones en aquesta vida toda, merezca alcanzar perdn de las unas y las otras.

30

55

MATERIAL FOTOCOPIABLE / Oxford University Press Espaa, S. A.

35

40

60

Pedro CALDERN DE LA BARCA El gran teatro del mundo Ctedra

1 Explica el contenido de este fragmento. 2 Qu caracteriza a los personajes? 3 Analiza la mtrica empleada en estos versos de El gran teatro del mundo.

4 Identifica las figuras estilsticas del texto y explica su funcin. 5 Qu caractersticas de los autos sacramentales observas en el fragmento?

ACTIVIDADE S

El burlador de Sevilla
El burlador de Sevilla, atribuida a Tirso de Molina, constituye una creacin dramtica original que, aunque utiliza componentes previos, crea uno de los personajes ms importantes de la literatura: el don Juan, que gener una importante descendencia. En este fragmento, tras engaar a una mujer noble, don Juan seduce a Tisbea, una pescadora, prometindole casamiento; pero en verdad piensa huir una vez consumados sus propsitos.

14

El engao de don Juan


DON JUAN.Mientras que los pescadores van de regocijo y fiesta, t las dos yeguas apresta1, que de sus pies voladores solo nuestro engao fo. CATALINN.Al fin pretendes gozar a Tisbea? DON JUAN.Si el burlar es hbito antiguo mo, qu me preguntas, sabiendo mi condicin? CATALINN. Ya s que eres castigo de las mujeres. DON JUAN.Por Tisbea estoy muriendo, que es buena moza. CATALINN. Buen pago a su hospedaje deseas. DON JUAN.Necio, lo mismo hizo Eneas2 con la reina de Cartago. CATALINN.Los que fings, y engais las mujeres de esa suerte, lo pagaris en la muerte. DON JUAN.Qu largo me lo fiis! Catalinn3 con razn te llaman. CATALINN. Tus pareceres sigue, que en burlar mujeres quiero ser Catalinn. Ya viene la desdichada. DON JUAN.Vete, y las yeguas prevn. CATALINN.Pobre mujer, harto bien te pagamos la posada. (Vase CATALINN y sale TISBEA.) 20 15 10 TISBEA.El rato que sin ti estoy estoy ajena de m. DON JUAN.Por lo que finges as ningn crdito te doy. TISBEA.Por qu? DON JUAN. Porque si me amaras mi alma favorecieras. TISBEA.Tuya soy. DON JUAN. Pues di, qu esperas, o en qu, seora, reparas? TISBEA.Reparo en que fue castigo de amor el que he hallado en ti. DON JUAN.Si vivo, mi bien, en ti a cualquier cosa me obligo, aunque yo sepa perder en tu servicio la vida, la diera por bien perdida, y te prometo de ser tu esposo. 40 35 30

45
Tirso DE MOLINA El burlador de Sevilla Ctedra

25

aprestar: aparejar, preparar, disponer lo necesario para algo.


MATERIAL FOTOCOPIABLE / Oxford University Press Espaa, S. A.

Eneas: legendario hroe de la guerra de Troya que abandon a la reina Dido para seguir rumbo a Italia, dejndola enamorada y desesperada. Catalinn: probablemente, cobarde.

1 Resume el contenido de estos versos de El burlador de Sevilla y relacinalo con lo que sabes sobre el personaje de don Juan. 2 Explica la actitud que mantiene el criado Catalinn ante su amo y la respuesta que este le da.

3 Qu mtrica emplea en estos versos Tirso de Molina? A qu tipo de estructura estrfica corresponde? 4 Analiza los principales recursos estilsticos presentes en este fragmento de El burlador de Sevilla.

Evaluacin 14

La dignidad del villano


JUAN.Que quieras, siendo t rico, vivir a estos hospedajes sujeto! PEDRO CRESPO. Pues cmo puedo excusarlos ni excusarme? JUAN.Comprando una ejecutoria. PEDRO CRESPO.Dime por tu vida, hay alguien que no sepa, que yo soy, si bien de limpio linaje, hombre llano? No por cierto. Pues, qu gano yo en comprarle una ejecutoria al Rey, si no le compro la sangre? Dirn entonces, que soy mejor que ahora? No, es dislate. Pues qu dirn? Que soy noble por cinco o seis mil reales; y esto es dinero y no es honra; que honra no la compra nadie. Quieres, aunque sea trivial, un ejemplillo escucharme? Es calvo un hombre mil aos, y al cabo de ellos se hace una cabellera. Este, en opiniones vulgares, deja de ser calvo? No. 25 Pues qu dicen al mirarle? Bien puesta la cabellera trae Fulano. Pues qu hace, si, aunque no le vean la calva, todos que la tiene saben? 30 JUAN.Enmendar su vejacin, remediarse de su parte, y redimir vejaciones del sol, del hielo y del aire. PEDRO CRESPO.Yo no quiero honor postizo, 35 que el defecto ha de dejarme en casa. Villanos fueron mis abuelos y mis padres; sean villanos mis hijos.
Pedro CALDERN DE LA BARCA El alcalde de Zalamea, Ctedra

10

15

20

1. En esta escena, Juan le propone a su padre Pedro Crespo


que compre una ejecutoria carta que exima de pagar tributos al rey , para evitar alojar a don lvaro. Resume los argumentos que esgrime el alcalde para negarse.

6. Cul era el orden que se segua normalmente en las


representaciones de los corrales de comedia? Caracteriza las piezas cortas que se representaban.

7. Seala los rasgos propios de los autos sacramentales. 8. Establece diferencias entre el teatro de Lope de Vega
y el de Caldern. Menciona obras de uno y otro.

2. Qu tema del teatro barroco observas en este dilogo? 3. Analiza la mtrica empleada en este fragmento y reconoce los principales recursos expresivos utilizados.

9. Cules son los principales hitos del teatro renacentista


que influyeron en el teatro barroco? Recuerda autores y obras del siglo & VI.

4. Enuncia los rasgos de la comedia nacional y explcalos.


Cules de ellos reconoces en el fragmento?

5. Explica las diferencias entre el teatro cortesano y las


representaciones de los corrales de comedia.

10. Redacta una exposicin sobre las manifestaciones


dramticas en la Edad Media.

Das könnte Ihnen auch gefallen